NR 511 Midterm

¡Supera tus tareas y exámenes ahora con Quizwiz!

Deanna, age 6, was bitten by a friend's dog. Her mother asks you if the child needs antirabies treatment. You tell her: 1."If the dog is a domestic pet that has been vaccinated, the wound should be cleaned and irrigated." 2."Antirabies treatment must be started immediately." 3."Rabies can be contracted only through the bites of wild animals." 4."Wait until you have observed the animal for 2 weeks to determine if it is rabid."

"If the dog is a domestic pet that has been vaccinated, the wound should be cleaned and irrigated." rationale: If the dog is a domestic pet that has been vaccinated, the wound should be washed thoroughly with soap and water and then treated like any other wound.

Susan states that her fiancé has been frostbitten on the nose while skiing and is fearful that it will happen again. What do you tell her? 1."Don't worry—as long as he gets medical help in the first few hours after being frostbitten again, he'll recover." 2."Once frostbitten, he should not go out skiing again." 3."If it should happen again, massage the nose with a dry hand." 4."Infarction and necrosis of the affected tissue can happen with repeated frostbite."

"Infarction and necrosis of the affected tissue can happen with repeated frostbite." rationale: Permanent tissue damage can occur with a second episode of frostbite involving the same skin surface.

Mario, a 17-year-old high school student, came to the office for evaluation. He is complaining of persistent sore throat, fever, and malaise not relieved by the penicillin therapy prescribed recently at the urgent care center. As the nurse practitioner, what would you order next? 1.A throat culture. 2.A Monospot test. 3.A rapid antigen test. 4. Thayer-Martin plate test.

A Monospot test rationale: If a client has a persistent sore throat, fever, and malaise not relieved by penicillin therapy, a Monospot test should be performed to rule out mononucleosis (Epstein-Barr virus)

Marlene, a 57-year-old cashier, comes to the clinic because she is unable to differentiate between sharp and dull stimulation on both sides of her face. You suspect: 1.Bell palsy. 2.A lesion affecting the trigeminal nerve. 3.A stroke—brain attack, cerebrovascular accident (CVA). 4.Shingles.

A lesion affecting the trigeminal nerve. rationale: A lesion affecting the sensory portion of the trigeminal nerve could be manifested by bilateral symptoms

What would you expect to see on an abdominal series that would lead toward a diagnosis of small-bowel obstruction? 1.Air-fluid levels. 2.A lead pipe colon. 3.Free air under the diaphragm. 4.Steeple sign.

Air-fluid levels. rationale: This finding on an abdominal x-ray is the hallmark of a small-bowel obstruction.

Jill, a 34-year-old bank teller, presents with symptoms of hay fever. She complains of nasal congestion, runny nose with clear mucus, and itchy nose and eyes. On physical assessment, you observe that she has pale nasal turbinates. What is your diagnosis? 1.Allergic rhinitis. 2.Viral rhinitis. 3.Nasal polyps. 4.Nasal vestibulitis from folliculitis

Allergic rhinitis rationale: The symptoms of hay fever, also called allergic rhinitis, are similar to those of viral rhinitis but usually persist and are seasonal in nature. When assessing the nasal mucosa, you will observe that the turbinates are usually pale or violaceous because of venous engorgement

What is the most important thing a person can do to maintain healthy skin and hopefully reduce wrinkles? 1.Keep well hydrated. 2.Use sunscreen with a sun protection factor (SPF) of at least 45. 3.Avoid smoking. 4.Use mild defatted or glycerin soap.

Avoid smoking. rationale: The most important thing a person can do to maintain healthy skin is not smoke. Smokers develop more wrinkles and have elastosis, decreased tissue perfusion and oxygenation, and an adverse exposure to free radicals on elastic tissue.

Martin, age 24, presents to the office with an erythematous ear canal and pain on manipulation of the auricle. He is on vacation and has been swimming daily at the resort. What is your diagnosis? 1.Acute otitis media. 2.Chronic otitis media. 3.External otitis. 4.Temporomandibular joint (TMJ) syndrome

External otitis rationale: With external otitis, there is pain, an erythematous ear canal, and usually a history of recent swimming

In a young child, unilateral purulent rhinitis is most often caused by: 1.A foreign body. 2.A viral infection. 3.A bacterial infection. 4.An allergic reaction

FB rationale: In a young child, unilateral purulent rhinitis is most often caused by a foreign body. The key word here is unilateral

Rose has gastroesophageal reflux disease (GERD). You know she misunderstands your teaching when she tells you she will: 1.Avoid coffee, alcohol, chocolate, peppermint, and spicy foods. 2.Eat smaller meals. 3.Have a snack before retiring so that the esophagus and stomach are not empty at bedtime. 4.Stop smoking.

Have a snack before retiring so that the esophagus and stomach are not empty at bedtime. rationale: Rose should not have a snack before retiring. Clients with GERD should be instructed to avoid coffee, alcohol, chocolate, peppermint, and spicy foods; eat smaller meals; stop smoking; remain upright for 2 hours after meals; elevate the head of the bed on 6- to 8-in blocks; and refrain from eating for 3 hours before retiring.

Which of the following is not a complication of untreated group A streptococcal pharyngitis? 1.Glomerulonephritis. 2.Rheumatic heart disease. 3.Scarlet fever. 4.Hemolytic anemia.

Hemolytic anemia rationale: This is a complication of mononucleosis

Marge, age 36, is planning to go skiing with her fiancé. He has warned her about frostbite, and she is wondering what to do if frostbite should occur. You know she's misunderstood the directions when she tells you which of the following? 1."I should remove wet footwear if my feet are frostbitten." 2."I should rub the area with snow." 3"I should apply firm pressure to the area with a warm hand." 4."I should place my hands in my axillae if my hands are frostbitten."

I should rub the area with snow rationale:Rubbing or massaging the frostbitten area, especially with snow, may cause permanent tissue damage.

You auscultate Julie's abdomen and hear a peritoneal friction rub. Which condition do you rule out? 1.Peritonitis. 2.A liver or spleen abscess. 3.A liver or spleen metastatic tumor. 4.Irritable bowel syndrome.

IBS rationale: IBS does not produce a friction rub

Sandy, age 52, presents with jaundice, dark urine, and light-colored stools, stating that she is slightly improved over last week's symptoms. Which stage of viral hepatitis do you suspect? 1.Incubation. 2.Prodromal. 3.Icteric. 4.Convalescent.

Icteric rationale: In the icteric stage of viral hepatitis, there is jaundice, dark urine, and light-colored stools. There are continued prodromal complaints, with gradual improvement.

A 25-year-old male presents complaining of hematochezia. The patient states he has noticed this for 2 days. He states there is a streak of bright blood along his stool every time he has a bowel movement. The patient has no pain with his bowel movements. He admits to eating a poor diet. The patient has no abdominal pain, nausea, or vomiting. On physical exam the patient has a positive fecal occult blood test but has no noticeable rectal bleeding or lesions. What is the likely diagnosis? 1.Internal hemorrhoid. 2.External hemorrhoid. 3.Bleeding peptic ulcer. 4.Rectal fissure.

Internal hemorrhoid. rationale: This is the likely diagnosis, as the patient has painless hematochezia.

Rose, a client with gastroesophageal reflux disease (GERD), has many other concurrent conditions. In teaching Rose about medications to avoid, what do you recommend she refrain from using? 1.Antibiotics. 2.Nonsteroidal anti-inflammatory drugs (NSAIDs). 3.Oral contraceptives. 4.Antifungals.

NSAIDS rationale: Clients with GERD should avoid taking NSAIDs because they tend to aggravate the already irritated gastric mucosa.

Marjorie, age 37, has asthma and has been told she has nasal polyps. What do you tell her about them? 1.Nasal polyps are usually precancerous. 2.Nasal polyps are benign growths. 3.The majority of nasal polyps are neoplastic. 4.They are probably inflamed turbinates, not polyps, because polyps are infrequent in clients with asthma

Nasal polyps are benign growths rationale: Nasal polyps are benign growths that occur frequently in clients with sinus problems, asthma, and allergic rhinitis. Polyps are neither neoplastic growths nor precancerous, but they do have the potential to affect the flow of air through the nasal passages. Clients who have asthma and have nasal polyps may have an associated allergy to aspirin, a syndrome that is referred to as Samter triad

Marjorie, age 37, has asthma and has been told she has nasal polyps. What do you tell her about them? 1.Nasal polyps are usually precancerous. 2.Nasal polyps are benign growths. 3.The majority of nasal polyps are neoplastic. 4.They are probably inflamed turbinates, not polyps, because polyps are infrequent in clients with asthma.

Nasal polyps are benign growths rationale: Nasal polyps are benign growths that occur frequently in clients with sinus problems, asthma, and allergic rhinitis. Polyps are neither neoplastic growths nor precancerous, but they do have the potential to affect the flow of air through the nasal passages. Clients who have asthma and have nasal polyps may have an associated allergy to aspirin, a syndrome that is referred to as Samter triad

During a camping trip, Jim, age 35, abruptly developed fever, headache, and joint pain. A few days after the onset of the fever, a blanchable macular rash began on his wrists and ankles and quickly spread to the palms and soles before becoming generalized. The rash is now petechial. You suspect Jim has: 1.Rocky Mountain spotted fever. 2.Flea bites. 3.Kawasaki disease. 4.Lyme disease.

Rocky Mountain spotted fever. rationale: A macular rash of the wrists and ankles that is followed by spread to the palms and soles before then becoming petechial is a characteristic finding of Rocky Mountain spotted fever.

Matt, age 26, recently returned from a camping trip and has gastroenteritis. He says that he has been eating only canned food. Which of the following pathogens do you suspect? 1.Campylobacter jejuni. 2.Clostridium botulinum. 3.Clostridium perfringens. 4.Staphylococcus.

Clostridium botulinum rationale: C botulinum is an anaerobic, gram-positive bacillus that produces toxins. It is widely distributed in the soil and vegetation. Improperly processed home-canned low-acid vegetables and contaminated meats are the usual cause of food-borne botulism

Which of the following is not a risk factor for cholecystitis? 1.Female gender. 2.Obesity. 3.Sickle cell anemia. 4.Younger age

Younger age rationale: The risk of cholecystitis actually increases with age

A 75-year-old African American male presents to your family practice office complaining of visual impairment. He has worn corrective lenses for many years but has noticed that his vision has gotten progressively worse the past 6 months. He denies pain. He states his vision is worse in both eyes in the peripheral aspects of his visual field. He also notes trouble driving at night and halos around street lights at night. You test his intraocular pressure, and it is 23 mm Hg. What is his most likely diagnosis? 1.Open-angle glaucoma. 2.Angle-closure glaucoma. 3.Cataracts. 4.Macular degeneration

open-angle glaucoma rationale: This is the typical presentation of chronic, or open-angle, glaucoma.

Which of the following is not recommended for hoarseness? 1.Vocal rest. 2.Tobacco cessation. 3.Decrease in caffeine use. 4.Oral steroids.

oral steroids rationale: Oral steroids are not routinely used to treat hoarseness

Samantha, age 12, presents with ear pain. When you begin to assess her ear, you tug on her normal-appearing auricle, eliciting severe pain. This leads you to suspect: 1.Otitis media. 2.Otitis media with effusion. 3.Otitis externa. 4.Primary otalgia.

otitis externa rationale: When severe pain is elicited by tugging on a normal-appearing auricle, an acute infection of the external ear canal (otitis externa) is suspected

Which of the following has/have not been linked to the use of isotretinoin? 1.Elevated liver transaminases. 2.Depression, psychosis, and suicidality. 3.Benign intracranial hypertension. 4.Pancreatitis.

pancreatitis rationale: This is not an adverse effect of isotretinoin

Margie, age 52, has an extremely stressful job and was just given a diagnosis of gastric ulcer. She tells you she is sure it is going to be malignant. How do you respond? 1."Don't worry. Gastric ulcers are not cancerous." 2."About 95% of gastric ulcers are benign." 3."You have about a 50% chance of having gastric cancer from your ulcer." 4."Even if it is cancer, surgery is 100% successful."

"About 95% of gastric ulcers are benign." rationale: About 95% of gastric ulcers are benign even though some of these seem to look malignant on x-ray.

Tina has a chronic hepatitis C infection. She asks you how to prevent its transmission. You respond: 1."Do not donate blood until one year after diagnosis." 2."Abstain from sex altogether." 3."There is no possibility of transmission through razors or toothbrushes." 4."Abstain from sex during your period."

"Abstain from sex during your period. rationale: Because the hepatitis C virus is transmitted in blood, including menstrual blood, clients should abstain from sex during menstruation. You might also test Tina to see which genotype her hepatitis C is to see if she is a candidate for Harvoni or Epclusa.

Ashley, age 6 months, has a Candida infection in the diaper area. What do you suggest to the parent? 1."Use rubber or plastic pants to contain the infection and prevent it from getting to the thighs." 2."Keep the area as dry as possible." 3."Use baby powder with cornstarch." 4."Keep Ashley away from other babies until the infection is cleared up.

"Keep the area as dry as possible." rationale: Clients must be taught to decrease favorable environmental conditions for Candida (eg, moisture, warmth, and poor air circulation)

Ashley, age 6 months, has a Candida infection in the diaper area. What do you suggest to the parent? 1."Use rubber or plastic pants to contain the infection and prevent it from getting to the thighs." 2."Keep the area as dry as possible." 3."Use baby powder with cornstarch." 4."Keep Ashley away from other babies until the infection is cleared up."

"Keep the area as dry as possible." rationale: Clients must be taught to decrease favorable environmental conditions for Candida (eg, moisture, warmth, and poor air circulation).

Nystatin (Mycostatin) is ordered for Michael, a 56-year-old banker who has an oral fungal infection. What instructions for taking the medication do you give Michael? 1."Dilute the oral medication with one tablespoon of water for easier digestion." 2."Take the medication with meals so that it's absorbed better." 3."Swish and swallow the medication." 4."Apply the medication only to the lesions.

"Swish and swallow the medication." rationale: When ordering nystatin (Mycostatin) for an oral fungal infection, tell the client to swish the medication in the mouth to coat all the lesions and then to swallow it.

Aaron, age 4, is brought in to the clinic by his father. His tympanic membrane is perforated from otitis media. His father asks about repair of the eardrum. How do you respond? 1."The eardrum, in most cases, heals within several weeks." 2."We need to schedule Aaron for a surgical repair." 3."He must absolutely stay out of the water for 3 to 6 months." 4."If the eardrum is not healed in several months, it can be surgically repaired."

"The eardrum, in most cases, heals within several weeks. rationale: Most perforated tympanic membranes seen with acute otitis media heal within several weeks

Your client's 2-month-old daughter is admitted with gastroenteritis and dehydration after 2 days of vomiting and diarrhea. When the father asks you what is causing the child's diarrhea, how do you respond? 1."She must be lactose intolerant from the formula, and this is altering the fluid balance." 2."Her body's telling you that it's time to introduce some solids into her system." 3."The virus is causing irritation of the gastrointestinal lining, which causes diarrhea." 4."The infectious agent invaded the stomach lining and is affecting the balance of water and nutrients."

"The infectious agent invaded the stomach lining and is affecting the balance of water and nutrients." rationale: In 80% of cases, gastroenteritis is viral in nature. This viral infection causes diarrhea by stimulating the secretion of electrolytes into the intestine. This is rapidly followed by water along the osmotic gradient, resulting in watery stools.

Marvin, a known alcoholic with cirrhosis, is frequently admitted for coagulopathies and occasionally receives blood transfusions. His wife asks you why he has bleeding problems. How do you respond? 1."Occasionally he accumulates blood in the gut." 2."There is an interruption of the normal clotting mechanism." 3."Long-term alcohol abuse has made his vessels very friable." 4."His bone marrow has been affected."

"There is an interruption of the normal clotting mechanism." rationale: Because of Marvin's alcoholism and his resulting dietary insufficiencies, there is an inadequate amount of vitamin K in the liver for thrombin to convert fibrinogen to fibrin; thus, the sequence of coagulation is disrupted.

Sally, age 21, is to undergo a tonsillectomy. She has heard about permanent taste changes after a tonsillectomy. What do you tell her? 1." As the tongue is responsible for sweet, sour, salty, and bitter taste abilities, they will all be affected somewhat." 2."You will have some alterations, but we'll have to wait and see how you are affected personally." 3."You may notice a slight difference initially, but there are no lasting changes in taste." 4."About half of the patients have some permanent alterations in the sense of taste."

"You may notice a slight difference initially, but there are no lasting changes in taste." rationale: Although some clients report a significant subjective drop in taste function following surgery, none have ongoing taste dysfunction.

Sandy asks what she can do for Dolores, her 90-year-old mother, who takes a bath every day and who has extremely dry skin. You respond: 1."After bathing every day, use a generous amount of moisturizer." 2."Use a special moisturizing soap every day." 3."Your mother does not need a bath every day." 4."Increase your mother's intake of fluids."

"Your mother does not need a bath every day." rationale: Dolores does not need a bath every day because that will exacerbate the dryness of her skin

Which human papillomavirus serotypes most commonly cause cancer? 1.Serotypes 16 and 18. 2.Serotypes 6 and 11. 3.Serotypes 3 and 10. 4.Serotypes 27 and 29

16 & 18 rationale: These serotypes can cause cancer

Icterus due to hyperbilirubinemia is seen when the serum level of bilirubin is greater than? 1.2.5 mg/dL. 2.1.0 mg/dL. 3.2.0 mg/dL. 4.0.5 mg/dL.

2.5 mg/dL. rationale: A bilirubin level greater than 2.5-3 mg/dL is associated with yellowing of the eyes.

When you are assessing the internal structure of the eye of your 59-year-old patient, the absence of a red reflex may indicate: 1.A cataract or hemorrhage into the vitreous humor. 2.Acute iritis. 3.Nothing; this is a normal finding in older adults. 4.Diabetes or long-standing hypertension

A cataract or hemorrhage into the vitreous humor. rationale: When assessing the internal structure of the eye, absence of a red reflex may indicate the total opacity of the pupil because of a cataract or hemorrhage into the vitreous humor. It may also be a result of improper positioning of the ophthalmoscope

Sharon, a 47-year-old bank teller, is seen by the nurse practitioner in the office for a red eye. You are trying to decide between a diagnosis of conjunctivitis and iritis. One distinguishing characteristic between the two is: 1.Eye discomfort. 2.Slow progression. 3.A ciliary flush. 4.No change in or slightly blurred vision

A ciliary flush rationale: When trying to decide between a diagnosis of conjunctivitis and iritis, one distinguishing characteristic is the ciliary flush present in iritis. Photophobia is not usually present in conjunctivitis, but it is always present with iritis. Photophobia occurs with corneal inflammation, iritis, and angle-closure glaucoma

Tom has just been diagnosed with celiac disease. Which of the following might you tell him? 1.There is a new pharmaceutical cure for celiac disease. 2.A strict gluten-free diet is the only treatment for celiac disease. 3.Your children will not be at a higher risk for developing this disease. 4.The presence of celiac disease is decreasing dramatically in the United States.

A strict gluten-free diet is the only treatment for celiac disease.

Mr. Johnson, age 69, has had Meniere disease for several years. He has some hearing loss but now has persistent vertigo. What treatment might be instituted to relieve the vertigo? 1.A labyrinthectomy. 2.Pharmacological therapy. 3.A vestibular neurectomy. 4.Wearing an earplug in the ear that has the most hearing loss

A vestibular neurectomy. rationale: For a client who has had Meniere disease for several years, with some hearing loss and now persistent vertigo, treatment by vestibular neurectomy might relieve the vertigo. In vestibular neurectomy, the portion of cranial nerve VIII that controls balance and the sensation of vertigo is severed. Vertigo is usually relieved in 90% of cases

Your client, a 72-year-old smoker of 50 years, is at the office today for a routine physical. During your inspection of the oral mucosa, you discover a white lesion on the lateral surface of the tongue that you suspect to be cancerous. You document your finding as: 1.A superficial, translucent, subepithelial, vesicle-like lesion in the oral mucosa. 2.A white, painless, firm, ulcerated lesion with indurated borders. 3.An abnormal white coating of the dorsal surface of the tongue. 4.A round, smooth, firm lump on the lateral side of the tongue

A white, painless, firm, ulcerated lesion with indurated borders rationale: Approximately 90% of oral cancers are squamous cell carcinoma (SCC), which is seen typically on the lip or lateral part of the tongue, usually as a lesion that is white, red, or mixed white and red. SCCs are characterized by painless, firm lesions with indurated borders

You suspect that Nikki has a gastroduodenal ulcer caused by Helicobacter pylori and plan to treat her empirically. What medications should you order? 1.A proton pump inhibitor (omeprazole), tetracycline or amoxicillin, and metronidazole (Flagyl). 2.Bismuth subsalicylate (Pepto-Bismol) and omeprazole (Prilosec). 3.Amoxicillin (Amoxil) and omeprazole (Prilosec). 4.Clarithromycin (Biaxin) and metronidazole (Flagyl)

Amoxicillin (Amoxil) and omeprazole (Prilosec) rationale: Both drugs listed are used in the eradication of H pylori, but do not complete the regimen.

A 6-year-old female presents to your pediatric office with her mother complaining of right ear pain for 3 days. This pain resolved with Tylenol. The patient has also had noted fevers of 101.3°F over the last 2 nights. The patient had a nonproductive cough for 7 days prior to the ear pain. On physical exam, the patient has tenderness with tugging on the auricle of the ear. The tympanic membrane is not mobile with pneumatic otoscopy and is erythematous and full. The patient has no drainage from the ear and no mastoid tenderness. What is the next step? 1.Symptom management and reassurance that symptoms will resolve with time. 2.Computed tomography (CT) of the head. 3.Amoxicillin 80 to 90 mg/kg/d. 4.Augmentin 45 mg/kg/d.

Amoxicillin 80 to 90 mg/kg/d. rationale: This is the treatment of choice for acute otitis media

What is the recommended treatment to eradicate a Helicobacter pylori infection? 1.Ranitidine, amoxicillin, and clarithromycin for 2 weeks. 2.Amoxicillin, clarithromycin, and omeprazole for 2 weeks. 3.Bismuth, amoxicillin, and clarithromycin for 2 weeks. 4.Bismuth, doxycycline, metronidazole, and ranitidine.

Amoxicillin, clarithromycin, and omeprazole for 2 weeks rationale: This is the recommended treatment to eradicate H pylori.

What is the best diagnostic test to confirm the diagnosis of celiac disease? 1.Anti-tTG IgA. 2.Anti-dsDNA. 3.Colonoscopy. 4.Anti-CCP protein.

Anti-tTG IgA. rationale: This is the most sensitive and specific test for diagnosing celiac disease.

All of the following medications are used for the control of nausea and vomiting. Which medication works by affecting the chemoreceptor trigger zone, thereby stimulating upper gastrointestinal motility and increasing lower esophageal sphincter pressure? 1.Anticholinergics, such as scopolamine (Donnatal). 2.Antidopaminergic agents, such as prochlorperazine (Compazine). 3.Antidopaminergic and cholinergic agents, such as metoclopramide (Reglan). 4.Tetrahydrocannabinols, such as dronabinol (Marinol)

Antidopaminergic and cholinergic agents, such as metoclopramide (Reglan) rationale: Metoclopramide (Reglan) is used for diabetic gastroparesis and postoperative nausea and vomiting. It works by affecting the chemoreceptor trigger zone, thereby stimulating upper gastrointestinal motility and increasing lower esophageal sphincter pressure

Bobby, age 6, has constant periumbilical pain shifting to the right lower quadrant, vomiting, a small volume of diarrhea, absence of headache, a mild elevation of the white blood cell count with an early left shift, and white blood cells in the urine. You suspect: 1.Appendicitis. 2.Gastroenteritis. 3.Acute pancreatitis. 4.Rocky Mountain spotted fever.

Appendicitis. rationale: Constant periumbilical pain shifting to the right lower quadrant; vomiting following the pain; a small volume of diarrhea; no systemic symptoms, such as a headache, malaise, or myalgia; a mild elevation of the white blood cell count with an early left shift; and white blood cells (WBCs) or red blood cells (RBCs) in the urine are indications of appendicitis. The WBC count becomes high only with gangrene or perforation of the appendix. The urine may have WBCs or RBCs if the bladder is irritated and ketonuria if there is prolonged vomiting.

A mother brings in her 4-year-old child, who she states has acute abdominal pain and a rash. Which of the following do you initially rule out? 1.Rocky Mountain spotted fever. 2.Measles. 3.Appendicitis. 4.A food allergy.

Appendicitis. rationale: There are many systemic causes of acute abdominal pain that also result in a rash. Appendicitis does not present with a rash.

Kathleen, age 54, has persistent pruritus of the external auditory canal. External otitis and dermatological conditions, such as seborrheic dermatitis and psoriasis, have been ruled out. What can you advise her to do? 1.Use a cotton-tipped applicator daily to remove all moisture and potential bacteria. 2.Wash daily with soap and water. 3.Apply mineral oil to counteract dryness. 4.Avoid topical corticosteroids.

Apply mineral oil to counteract dryness. rationale: Pruritus of the external ear canal is a common problem. In most cases, the pruritus is self-induced from overenthusiastic cleaning or excoriation. The protective cerumen covering must be allowed to regenerate and may be helped to do so by application of a small amount of mineral oil, which helps counteract dryness and reject moisture. Often, the use of isopropyl alcohol may relieve ear canal pruritus as well

Susan, age 59, has no specific complaints when she comes in for her annual examination. She does, however, have type 2 diabetes mellitus (DM), slight hypertension, dyslipidemia, and central obesity. How would you diagnose her? 1.As a healthy adult with several problems. 2.As having a glycemic event. 3.As having metabolic syndrome. 4.As having multiple organ dysfunction.

As having metabolic syndrome rationale: Susan has a constellation of symptoms known as metabolic syndrome. The World Health Organization (WHO), National Cholesterol Education Program Adult Treatment Panel III (NCEP ATP III), and International Diabetes Federation (IDF) have slightly different criteria for this diagnosis. They all, however, include hypertension, dyslipidemia, and central obesity.

The ABCDEs of melanoma identification include which of the following? 1.Asymmetry: one half does not match the other half. 2.Border: the borders are regular; they are not ragged, notched, or blurred. 3.Color: pigmentation is uniform. 4.Diameter: the diameter is 5 mm

Asymmetry: one half does not match the other half. rationale: A is for asymmetry: one half does not match the other half. One of the warning signs of cancer is a lesion that does not heal or an area that changes in appearance. The ABCDEs of melanoma identification should be taught to all clients.

The ABCDEs of melanoma identification include which of the following? 1.Asymmetry: one half does not match the other half. 2.Border: the borders are regular; they are not ragged, notched, or blurred. 3.Color: pigmentation is uniform. 4.Diameter: the diameter is 5 mm.

Asymmetry: one half does not match the other half. rationale: A is for asymmetry: one half does not match the other half. One of the warning signs of cancer is a lesion that does not heal or an area that changes in appearance. The ABCDEs of melanoma identification should be taught to all clients.

Candidiasis may occur in many parts of the body. James, age 29, has it in the glans of his penis. What is your diagnosis? 1.Balanitis. 2.Thrush. 3.Candidal paronychia. 4.Subungual Candida

Balanitis rationale: Candidiasis of the glans of the penis is balanitis

Treatment for achalasia may include: 1.Balloon dilation of the lower esophageal sphincter. 2.Beta blockers. 3.A fundoplication. 4.An esophagogastrectomy.

Balloon dilation of the lower esophageal sphincter. rationale: Achalasia is an absence of peristalsis of the esophagus and a high gastroesophageal sphincter pressure. After initial noninvasive treatments, clients may require a balloon dilation of the lower esophageal sphincter.

Which skin cancer that arises from skin cells, characteristically occurs on body areas exposed to the sun, most commonly presents as a pearly nodule with fine telangiectasias over the surface and a border that appears rolled, and is the most common skin cancer? 1.Actinic keratosis. 2.Basal cell carcinoma. 3.Squamous cell carcinoma. 4.Melanoma.

Basal cell carcinoma. rationale: Basal cell carcinoma arises from skin cells and is the most common type of nonmelanoma skin cancer (greater than 80%).

Sandra, age 69, is complaining of dry skin. What do you advise her to do? 1.Every day, when bathing, vigorously use a washcloth to exfoliate the upper layers of the stratum corneum. 2.Bathe or shower with lukewarm water and use a mild soap or skin cleanser. 3.Use a dehumidifier. 4.Decrease the oral intake of fluids.

Bathe or shower with lukewarm water and use a mild soap or skin cleanser. rationale: If a client is complaining of dry skin, the client should use tepid water and a mild cleansing cream or soap.

Shelby has recently been diagnosed with pancreatitis. Of the following objective findings that can result from the pancreatic inflammatory process, which is known as Grey Turner sign? 1.Left-sided pleural effusion. 2.Bluish discoloration over the flanks. 3.Bluish discoloration around the umbilicus. 4.Jaundice.

Bluish discoloration over the flanks. rationale: Grey Turner sign is a bluish discoloration over the flanks.

A 45-year-old homeless man presents to your urgent care clinic for evaluation. His chief complaint is diarrhea. The patient states he started to have diarrhea 2-3 days ago, and it is getting progressively worse. He also notes nausea without vomiting, dry mouth, and double vision. On exam you notice his pupillary reflex is absent. The patient states he lives on the street and eats mostly canned goods that he scavenges from a grocery store dumpster. What is the likely cause of the patient's symptoms? 1.Botulism. 2.Salmonella. 3.Lyme disease. 4.Vitamin C deficiency.

Botulism. rationale: This patient has a gastrointestinal illness associated with descending neurological symptoms after eating canned food; this is the presentation of botulism.

Shelby, age 14, has a blister on her arm that is filled with clear fluid. It is the result of contact with a hot iron. How do you document this? 1.Bulla. 2.Wheal. 3.Cyst. 4.Pustule

Bulla rationale: A bulla is a primary skin lesion that is filled with fluid and larger than 1 cm in diameter. It is also known as a vesicle.

Justin, an obese 42-year-old, cut his right leg 3 days ago while climbing a ladder. Today his right lower leg is warm, reddened, and painful, without a sharply demarcated border. What do you suspect? 1.Diabetic neuropathy. 2.Cellulitis. 3.Peripheral vascular disease. 4.A beginning stasis ulcer.

Cellulitis rationale:Cellulitis is a spreading infection of the epidermis and subcutaneous tissue that usually begins after a break in the skin. The skin of this patient's right lower leg is warm, red, and painful. Although Justin may have diabetic neuropathy, peripheral vascular disease, or a stasis ulcer, the information is not complete enough for you to suspect those conditions. The information and assessment data given fully support a diagnosis of cellulitis.

Ruby has a colostomy and complains that her stools are too loose. What food(s) do you suggest to help thicken the stools? 1.Cheese. 2.Leafy green vegetables. 3.Raw fruits and vegetables. 4.Dried beans.

Cheese rationale: Cheese, bread, pasta, rice, pretzels, and yogurt all help to thicken stools.

Marian, age 52, is obese. She complains of a rapid onset of severe right upper quadrant abdominal cramping pain, nausea, and vomiting. Your differential diagnosis might be: 1.Appendicitis. 2.Crohn's disease. 3.Cholecystitis. 4.Irritable bowel syndrome.

Cholecystitis. rationale: A rapid onset of severe right upper quadrant (RUQ) abdominal cramping pain with nausea and vomiting is a classic presentation of acute cholecystitis; 90% to 95% of clients with acute cholecystitis also have gallstones. Other symptoms include low-grade fever, epigastric tenderness, guarding, and pain on inspiration during palpation of the RUQ (Murphy sign). The 7 F's of cholecystitis are fair, fat, 40, female, fertile, fat intolerant, and flatulent.

The screening guidelines for colon cancer recommend which of the following for the general population? 1.Colonoscopy starting at age 50. 2.Colonoscopy starting at age 45. 3.Fecal occult blood test (FOBT) and rectal exam starting at age 50. 4.Fecal occult blood test (FOBT) and rectal exam starting at age 45.

Colonoscopy after age 50 rationale: This is the recommended age to start colonoscopic examination in someone with a low risk for colon cancer

Timothy, age 68, complains of an abrupt change in his defecation pattern. You evaluate him for: 1.Constipation. 2.Colorectal cancer. 3.Irritable bowel syndrome. 4.Acute appendicitis.

Colorectal cancer rationale: A middle-aged or older client with an abrupt change in defecation pattern must be evaluated for colorectal cancer

A 50-year-old female presents to the urgent care clinic complaining of left lower quadrant pain. She has associated nausea and vomiting, and her vital signs are as follows: temperature 102.5°F, pulse 110, blood pressure 150/90, pulse oximetry 99% on room air. What is the best test to evaluate this patient? 1.Sigmoidoscopy. 2.Abdominal series. 3.Computed tomography (CT) scan with oral contrast. 4.Abdominal ultrasound.

Computed tomography (CT) scan with oral contrast rationale: This is the best diagnostic choice. While diverticulitis is likely the cause of the patient's symptoms, a CT of the abdomen would show if the patient has any gynecologic etiology (such as ovarian cyst or tumor) of this pain

A 16-year-old male presents to your office. He was sent by an orthopedist. He has recently had surgical fixation of a humerus fracture. The patient has been going to physical therapy and has been developing a rash on his arm after therapy that disappears shortly after returning home. He does not have the rash prior to therapy. The patient denies fevers and chills, and his incision is well healed, with no signs of infection. Of note, the patient has been experiencing more hand edema than the average patient and has had edema wraps used at the end of therapy to help with his swelling. The wraps are made of a synthetic plastic material. The rash the patient gets is erythematous and blotchy, not raised; it is on the operative upper extremity. What is the most likely diagnosis? 1.Contact dermatitis. 2.Atopic dermatitis. 3.Seborrheic dermatitis. 4.Psoriasis.

Contact dermatitis. rationale: The patient's history and rash are consistent with a latex or plastic sensitivity due to the edema wraps used in therapy.

Which disease usually starts on the cheeks and spreads to the arms and trunk? 1.Erythema infectiosum (fifth disease). 2.Rocky Mountain spotted fever. 3.Rubeola. 4.Rubella

Fifths disease rationale: Erythema infectiosum (fifth disease) usually starts on the cheeks and spreads to the arms and trunk.

You are counseling Lillian, who is lactose intolerant, about foods to avoid. You know she misunderstands the teaching when she tells you she can have: 1. Yogurt. 2.Foods containing whey. 3.Prehydrolyzed milk. 4.Oranges.

Foods containing whey rationale: Advise clients who are lactose intolerant to avoid foods containing whey. Whey is a lactose-rich ingredient found in some foods, so clients who are lactose intolerant need to read labels on all foods. To control symptoms, dietary lactose should be reduced or restricted by using lactose-reduced and lactose-free dairy products or by eating lactose-rich foods in small amounts or in combination with low-lactose or lactose-free foods.

Which of the following is the most common cause of acute pancreatitis? 1.Gallstone obstruction of the pancreatic duct. 2.Alcoholism. 3.Hypertriglyceridemia. 4.Gallstone obstruction of the pancreatic duct and alcoholism.

Gallstone obstruction of the pancreatic duct and alcoholism. rationale: Both gallstones and alcoholism are the most common causes of acute pancreatitis.

A 64-year-old obese woman comes in complaining of difficulty swallowing for the past 3 weeks. She states that "some foods get stuck" and she has been having "heartburn" at night when she lies down, especially if she has had a heavy meal. Occasionally, she awakes at night coughing. She denies weight gain and/or weight loss, vomiting, or change in bowel movements or color of stools. She denies alcohol and tobacco use. There is no pertinent family history or findings on review of systems (ROS). Physical examination is normal, with no abdominal tenderness, and the stool is occult blood (OB) negative. What is the most likely diagnosis? 1.Esophageal varices. 2.Esophageal cancer. 3.Gastroesophageal reflux disease (GERD). 4.Peptic ulcer disease (PUD)

Gastroesophageal reflux disease (GERD) rationale: Though the historical data are incomplete, this client has no obvious risk factors for esophageal varices or esophageal cancer. She is a nondrinker and denies weight loss and changes in bowel function or color of stools, which could be a clue to a gastrointestinal bleed. The fact that her worst symptoms occur at night with regurgitation and heartburn is classic for GERD. Dysphagia is frequently a prominent symptom of GERD. She has no abdominal tenderness, and aside from the nighttime symptoms and dysphagia, she reports no symptoms with food or lack of food

You are assessing a first grader and find that the tonsils are touching the uvula. How would you grade this finding? 1.Grade 1. 2.Grade 2. 3.Grade 3. 4.Grade 4

Grade 3 rationale: Grade 3 indicates the tonsils are touching the uvula. Tonsils are enlarged to 2, 3, or 4 with an acute infection

A mother complains that her newborn infant, while lying on his side, appears red on the dependent side of the body and pale on the upper side. When she picks up the baby, this coloring disappears. You explain to her about which of the following? 1.A temporary hemangioma. 2.Hyperbilirubinemia. 3.Harlequin sign. 4.Mongolian spots.

Harlequin's sign rationale: The harlequin sign is a transient phenomenon in a newborn who has been lying on one side. The dependent side is red while the upper side is pale, as if a line has been drawn down the middle of the body. This disappears when the infant's position is changed

In a burn trauma, which blood measurement rises as a secondary result of hemoconcentration when fluid shifts from the intravascular compartment? 1.Hemoglobin. 2.Sodium. 3.Hematocrit. 4.Blood urea nitrogen (BUN).

Hcrit rationale: In burn trauma, the hematocrit rises as fluid, not blood, shifts from the intravascular compartment.

Dan, age 57, has just been given a diagnosis of herpes zoster. He asks you about exposure to others. You tell him: 1.Once he has been on the medication for a full 24 hours, he is no longer contagious. 2.He should stay away from children and pregnant women who have not had chickenpox. 3.He should wait until the rash is completely gone before going out in crowds. 4.He should be isolated from all persons except his wife.

He should stay away from children and pregnant women who have not had chickenpox. rationale: If a client has just been given a diagnosis of herpes zoster, advise the client to stay away from children and pregnant women who have not had chickenpox until crusts have formed over the blistered areas.

Hepatitis D is an RNA virus that requires a coinfection with which of the following strains of hepatitis in order to replicate? 1.Hepatitis A. 2.Hepatitis B. 3.Hepatitis C. 4.Hepatitis E.

Hepatitis B. rationale: Hepatitis D requires hepatitis B surface antigen to replicate.

Regular ocular pressure testing is indicated for older adults taking: 1.High-dose inhaled glucocorticoids. 2.Nonsteroidal anti-inflammatory drugs (NSAIDs). 3.Angiotensin-converting enzyme (ACE) inhibitors. 4.Insulin

High-dose inhaled glucocorticoids. rationale: Although regular ocular pressure testing is indicated on a routine basis for all older adults, it is especially important for clients taking an extended regimen of high-dose inhaled glucocorticoids because prolonged continuous use increases the risk of ocular hypertension or open-angle glaucoma.

Which of the following is not true regarding hepatitis C? 1.The greatest rate of infection in the general population is seen in people born between 1945 and 1965. 2.Many hepatitis C infections are asymptomatic. 3.Hepatitis C is seen more frequently in men than women. 4.If hepatitis is asymptomatic it doesn't cause cirrhosis or liver cancer.

If hepatitis is asymptomatic it doesn't cause cirrhosis or liver cancer. rationale: This is untrue; the reason many people end up with cirrhosis or liver cancer is because the disease can often be asymptomatic.

You are doing routine teaching with a patient who has a family history of colorectal cancer. You know she misunderstands the teaching when she tells you she will: 1.Decrease her fat intake. 2.Increase her fiber intake. 3.Continue her daily use of aspirin. 4.Increase her fluid intake.

Increase her fluid intake. rationale: Increasing fluid intake has not been shown to decrease the risk of colorectal cancer. Current recommendations to aid in preventing colorectal cancer include decreased fat consumption, increased fiber consumption, and the daily use of aspirin.

You suspect that Harry has a peptic ulcer and tell him that it has been found to be strongly associated with: 1.Anxiety and panic attacks. 2.Long-term use of nonsteroidal anti-inflammatory drugs (NSAIDs). 3.Infection by Helicobacter pylori. 4.A family history of peptic ulcers.

Infection by H. Pylori rationale: About 90% of cases of peptic ulcers have been found to be caused by infection with the bacteria H pylori

You suspect that Harry has a peptic ulcer and tell him that it has been found to be strongly associated with: 1.Anxiety and panic attacks. 2.Long-term use of nonsteroidal anti-inflammatory drugs (NSAIDs). 3.Infection by Helicobacter pylori. 4.A family history of peptic ulcers

Infection by Helicobacter pylori. rationale: About 90% of cases of peptic ulcers have been found to be caused by infection with the bacteria H pylori.

Melva, age 63, presents with an acute exacerbation of pancreatitis, and you are going to admit her to the hospital. Which is the most important factor in determining a negative long-term outcome for her? 1.Age. 2.Infection. 3.Pain. 4.Length of time between exacerbations

Infection. rationale: The most important factor in determining long-term negative outcomes for pancreatitis is the presence of infection. Despite best practices, mortality associated with severe acute pancreatitis remains approximately 20% to 25% because of systemic complications.

You elicit costovertebral angle tenderness in Gordon, age 29. Which condition do you suspect? 1.Cirrhosis. 2.Inflammation of the kidney. 3.Inflammation of the spleen. 4.Peritonitis.

Inflammation of the kidney. rationale: Costovertebral angle tenderness is tenderness or sharp pain that is elicited when one hand is "thumped" with the ulnar edge of the other fist over the 12th rib at the costovertebral angle on the back. It indicates inflammation of the kidney (and possible associated renal calculi, renal artery or vein occlusion, and perirenal abscess).

Stacy, a nursing student, is to begin her series of hepatitis B vaccinations. You test her for a serological marker, and the results show hepatitis B surface antibodies (HBsAb). You tell Stacy that she: 1.Needs to begin the hepatitis B series as soon as possible. 2.Needs to be tested again because one reading is not indicative of immunity. 3.Is permanently immune to hepatitis B. 4.Has an acute hepatitis B infection

Is permanently immune to hepatitis B. rationale: The marker for permanent immunity, hepatitis B surface antibodies in the serum, will be present 4 to 10 months after exposure and immunity to hepatitis B.

Jonathan, age 19, has just been given a diagnosis of mononucleosis. Which of the following statements is true? 1.The offending organism is a bacterium, and Jonathan should be treated with antibiotics. 2.Convalescence is usually only a few days, and Jonathan should be back to normal in a week. 3.Mononucleosis is rarely contagious. 4.Jonathan should avoid contact sports and heavy lifting.

Jonathan should avoid contact sports and heavy lifting. rationale: When teaching clients about mononucleosis, or Epstein-Barr virus (EBV), tell them to avoid contact sports and heavy lifting because of splenomegaly and a threat of rupture.

Lee brings her 13-year-old son to your clinic. He has been complaining of a rash on the buttocks, anterior thighs, and posterolateral aspects of his upper arms. He tells you it is mildly pruritic and looks like "gooseflesh." On examination, the rash appears as small, pinpoint, follicular papules on a mildly erythematous base. You explain to Lee that the benign condition is likely to resolve by the time her son reaches adulthood, and it is known as: 1.Comedones of acne. 2.Molluscum contagiosum. 3.Keratosis pilaris. 4.Atopic dermatitis.

Ketatosis pilaris rationale: The description and examination of this rash are consistent with keratosis pilaris, which most commonly appears on the cheeks, buttocks, anterior thighs, and posterolateral aspects of the upper arms

Eric, age 52, has gout. What do you suggest? 1.Using salicylates for an acute attack. 2.Limiting consumption of purine-rich foods. 3.Testing his uric acid level every 6 months. 4.Decreasing fluid intake.

Limiting consumption of purine-rich foods rationale: For a client with gout, the consumption of purine-rich foods, such as organ meats, should be limited to prevent uric acid buildup

Eric, age 52, has gout. What do you suggest? 1.Using salicylates for an acute attack. 2.Limiting consumption of purine-rich foods. 3.Testing his uric acid level every 6 months. 4.Decreasing fluid intake.

Limiting consumption of purine-rich foods. rationale: For a client with gout, the consumption of purine-rich foods, such as organ meats, should be limited to prevent uric acid buildup.

Cydney has been given a diagnosis of ascariasis. Which symptoms would you expect to see? 1.Low-grade fever, productive cough with blood-tinged sputum, wheezing, and dyspnea. 2.Nocturnal perianal and perineal pruritus. 3.Diarrhea, cramps, and malaise. 4.Ascites and facial and extremity edema.

Low-grade fever, productive cough with blood-tinged sputum, wheezing, and dyspnea. rationale: Ascariasis is the most common intestinal helminth (parasitic worm). It causes pulmonary manifestations, such as low-grade fever, productive cough with blood-tinged sputum, wheezing, and dyspnea, because the larvae are transmitted to the lungs from the vascular system. The larvae burrow through alveolar walls, migrating up the bronchial tree to the pharynx, and then down the esophagus, back to the intestine.

Which treatment is considered the gold standard in tissue-conserving skin cancer removal? 1.Cryosurgery. 2.Simple excision. 3.Photodynamic therapy. 4.Mohs micrographic surgery (MMS).

MMS rationale: MMS is considered the gold standard in tissue-conserving skin cancer removal. MMS is a specialized type of surgery consisting of the removal of the entire tumor with the smallest possible margin of normal skin

Which of the following statements about macular degeneration is not true? 1.Macular degeneration is characterized by gradual loss of peripheral vision. 2.Macular degeneration is the leading cause of blindness in people younger than 60. 3.Tobacco use is a risk factor for macular degeneration. 4.There are 2 different types of macular degeneration: wet and dry

Macular degeneration is characterized by gradual loss of peripheral vision rationale: This is how open-angle glaucoma is characterized. Macular degeneration is gradual loss of central vision

A 20-year-old male presents to your primary care clinic. This patient is a college student. He complains of fatigue, sore throat, and low-grade fever for 3 days. On physical exam, he has a temperature of 100.7°F. His ear exam is normal. His nose and throat exam shows mild erythema of the nasal mucosa and edematous, enlarged tonsils bilaterally, with erythema of the pharyngeal wall and tonsillar exudates. He has inflamed posterior cervical lymph nodes. He has a mild nonproductive cough and clear lung exam. What is his most likely diagnosis? 1.Viral pharyngitis. 2.Mononucleosis. 3.Streptococcal pharyngitis. 4.Upper respiratory infection

Mono rationale: This presentation could be a viral pharyngitis; however, with posterior cervical lymphadenitis, you would suspect mononucleosis.

How would you describe the cervical lymphadenopathy associated with asymptomatic human immunodeficiency virus (HIV) infection? 1.Movable, discrete, soft, and nontender lymph nodes. 2.Enlarged, warm, tender, and firm but freely movable lymph nodes. 3.Hard, unilateral, nontender, and fixed lymph nodes. 4.Nontender, mobile, and firm but not hard lymph nodes

Nontender, mobile, and firm but not hard lymph nodes rationale: The cervical lymphadenopathy associated with asymptomatic HIV infection may be described as lymph nodes that are nontender, mobile, and firm but not hard

Mary, age 82, presents with several eye problems. She states that her eyes are always dry and look "sunken in." What do you suspect? 1.Hypothyroidism. 2.Normal age-related changes. 3.Cushing syndrome. 4.A detached retina

Normal age-related changes rationale: Dryness of the eyes and the appearance of "sunken" eyes are normal age-related changes

You suspect appendicitis in Andrew, who is 18. With his right hip and knee flexed, you slowly rotate his right leg internally to stretch a muscle. He states that it is painful over his right lower quadrant. Which sign did you elicit? 1.Rovsing sign. 2.Psoas sign. 3.Obturator sign. 4.McBurney sign

Obturator sign. rationale: Obturator sign is elicited when, with the patient's right hip and knee flexed, the examiner slowly rotates the right leg internally, which stretches the obturator muscle. Pain over the right lower quadrant (RLQ) is considered a positive sign.

Lucy, age 49, has pain in both the left and right lower quadrants. What might you suspect? 1.A gastric ulcer. 2. Gastritis .3.Pelvic inflammatory disease. 4.Pancreatitis.

PID rationale: The pain associated with pelvic inflammatory disease can be palpated in both the right and left lower quadrants.

Lucy, age 49, has pain in both the left and right lower quadrants. What might you suspect? 1.A gastric ulcer. 2.Gastritis. 3.Pelvic inflammatory disease. 4.Pancreatitis.

PID rationale: The pain associated with pelvic inflammatory disease can be palpated in both the right and left lower quadrants.

A 4-year-old male presents to your pediatric clinic with his mother complaining of an itchy rash, mostly between his fingers. This has been going on for multiple days and has been getting worse. The patient recently started at a new day care. On physical exam, the patient is afebrile and has multiple small (1-2 mm) red papules in sets of 3 located in the web spaces between his fingers. He also has signs of excoriation. What is the treatment for this problem? 1.Permethrin lotion for the patient and also his family members. 2.Cold compresses and hydrocortisone cream 1% twice a day. 3.Over-the-counter Benadryl cream. 4. Ketoconazole cream

Permethrin lotion for the patient and also his family members. rationale: This is the treatment for scabies.

A 7-year-old male presents with his mother to the urgent care clinic complaining of abdominal pain. He started to complain of pain prior to going to bed; however, it has gotten progressively worse and is now preventing him from sleeping. He is nauseous but hasn't vomited and didn't eat dinner due to the pain. The patient appears pale and is complaining of right-sided abdominal pain. His vitals are as follows: blood pressure 130/85, pulse 120, temperature 100.5°F, pulse oximetry 98% on room air. On physical exam he is tender in the right lower quadrant. His complete blood count (CBC) shows a white blood cell count (WBC) of 17.0. What is the patient's likely diagnosis? 1.Appendicitis. 2.Cholecystitis. 3.Constipation. 4.Gastroenteritis

appendicitis rationale: This is the clinical scenario typical for appendicitis

Which of the following is not a cause of conductive hearing loss? 1.Presbycusis. 2.Cerumen impaction. 3.Otitis media. 4.Otosclerosis

Presbycusis rationale: This is a cause of sensorineural hearing loss

Max, age 35, states that he thinks he has an ear infection because he just flew back from a business trip and feels unusual pressure in his ear. You diagnose barotrauma. What is your next action? 1.Prescribe nasal steroids and oral decongestants. 2.Prescribe antibiotic ear drops. 3.Prescribe systemic antibiotics. 4.Refer Max to an ear, nose, and throat specialist

Prescribe nasal steroids and oral decongestants. rationale: Barotrauma of the auditory canal, causing a sensation of abnormal middle ear pressure, may be relieved by the use of nasal steroids and oral decongestants.

Erica, age 39, has a sudden onset of shivering, sweating, headache, aching in the orbits, and general malaise and misery. Her temperature is 102°F. The nurse practitioner diagnosed her with influenza (flu). What is your next course of action? 1.Order amoxicillin (Amoxil) 500 mg every 12 hours for 7 days. 2.Prescribe rest, fluids, acetaminophen (Tylenol), and possibly a decongestant and an antitussive. 3.Order a complete blood count (CBC). 4.Consult with your collaborating physician

Prescribe rest, fluids, acetaminophen (Tylenol), and possibly a decongestant and an antitussive. rationale: Management of flu is generally symptomatic and includes rest, fluids, acetaminophen (Tylenol), and possibly a decongestant and an antitussive. The client should be advised to call or return to the clinic in 4 days if symptoms are not improving

Mr. Swanson, age 67, presents to the clinic for his annual health exam. He asks you if there is anything he can do to prevent the painful, blistering sores that develop on his lip in the summertime when he plays golf. You explain to Mr. Swanson that the way to prevent the development of these lesions is to: 1.Protect the lips from sun exposure with a blocking agent, such as zinc oxide, or a lip balm that contains a broad-spectrum sunscreen. 2.Apply acyclovir 5% cream 5 times a day for 4 days. 3.Take acyclovir 500 mg 1 tablet 5 times a day for 5 days. 4.Wear a visor

Protect the lips from sun exposure with a blocking agent, such as zinc oxide, or a lip balm that contains a broad-spectrum sunscreen. rationale: Mr. Swanson has recurrent herpes simplex virus type 1 (HSV-1), ie, orolabial herpes. Factors that trigger reactivation include local skin trauma, sunlight exposure, and systemic changes, such as menses, fatigue, and fever. In this question, the clinician is teaching prevention. Protecting the lips from sun exposure is a preventive measure

A 22-year-old college student presents to your urgent care clinic complaining of a rash. She was recently on spring break and spent every night in the hot tub at her hotel. On physical exam, she has multiple small areas of 1- to 2-mm erythematous pustules that are present mostly where her bathing suit covered her buttocks. What is the most likely pathogen causing these lesions? 1.Pseudomonas aeruginosa. 2.Klebsiella. 3.Staphylococcus aureus. 4.Streptococcus

Pseudomonas aeruginosa rationale: This is a common cause of hot tub folliculitis.

A 22-year-old college student presents to your urgent care clinic complaining of a rash. She was recently on spring break and spent every night in the hot tub at her hotel. On physical exam, she has multiple small areas of 1- to 2-mm erythematous pustules that are present mostly where her bathing suit covered her buttocks. What is the most likely pathogen causing these lesions? 1.Pseudomonas aeruginosa. 2.Klebsiella. 3.Staphylococcus aureus. 4.Streptococcus.

Pseudomonas aeruginosa. rationale: This is a common cause of hot tub folliculitis.

Which of the following statements about psoriasis is not true? 1.Psoriatic lesions are often silvery scales that form over erythematous plaques. 2.Psoriatic lesions often occur in the folds of the elbows and behind the knees. 3.People with psoriasis have a greater risk of depression than the average population. 4.Psoriasis has a genetic component.

Psoriatic lesions often occur in the folds of the elbows and behind the knees. rationale: This is untrue; lesions usually occur on the fronts of the knees, the posterior aspects of the elbows, and the scalp.

A 25-year-old male presents with "bleeding in my eye" for 1 day. He awoke this morning with a dark area of redness in his eye. He has no visual loss or changes. He denies constitutional symptoms, pruritus, drainage, or recent trauma. The redness presents on physical exam as a dark red area in the patient's sclera of the right eye only and takes up less than 50% of the eye. The patient's remaining sclera is clear and white. He also notes he was drinking alcohol last night and vomited afterward. What is the best treatment? 1.Topical steroids and close follow-up with an ophthalmologist. 2.Sending the patient to the emergency department for immediate ophthalmology consult. 3.Reassurance that this lesion will resolve without any treatment in 2 to 4 weeks. 4.Cold compresses and frequent handwashing

Reassurance that this lesion will resolve without any treatment in 2 to 4 weeks. rationale: This is the classic presentation of a subconjunctival hemorrhage. It will resolve without treatment in 2 to 4 weeks. Vomiting probably caused his hemorrhage

You suspect a platelet abnormality in a 40-year-old woman who presents to your clinic with: 1.Red to blue macular plaques. 2.Multiple freckle-like macular lesions in sun-exposed areas. 3.Numerous small, brown, nonscaly macules that become more prominent with sun exposure. 4.Red, flat, nonblanchable petechiae.

Red, flat, nonblanchable petechiae rationale: A client with a platelet abnormality may present with red, flat, nonblanchable petechiae.

Mattie, age 64, presents with blurred vision in 1 eye and states that it felt like "a curtain came down over my eye." She doesn't have any pain or redness. What do you suspect? 1.Retinal detachment. 2.Acute angle-closure glaucoma. 3.Open-angle glaucoma. 4.Cataract

Retinal detachment. rationale: The classic sign of retinal detachment is a client stating that "a curtain came down over my eye." Typically, the person presents with blurred vision in 1 eye that becomes progressively worse, with no pain or redness.

You are trying to differentiate between functional (acquired) constipation and Hirschsprung disease in a neonate. Distinguishing features of Hirschsprung disease include which of the following? 1.Small ribbonlike stools. 2.Obvious abdominal pain. 3.Female gender. 4.Small weight gain

Small ribbonlike stools. rationale: Hirschsprung disease is common in male infants, results in small ribbonlike stools, usually has no accompanying abdominal pain unless there is obstruction, and may be accompanied by failure to thrive. Other symptoms may include swollen belly; vomiting, including vomiting a green or brown substance; constipation; gas, which might make a newborn fussy; and diarrhea.

A 55-year-old landscaper presents to your primary care office complaining of a small skin lesion on his face. The patient states the lesion causes no pain or other symptoms. On physical exam, you notice a small (3 mm) papule that is flesh-colored and irregular. To palpation, the lesion feels hard and like sandpaper. What type of malignancy is this patient at risk for given the appearance of this lesion? 1.Squamous cell carcinoma. 2.Melanoma. 3.Basal cell carcinoma. 4.Rosacea.

Squamous cell carcinoma. rationale: The lesion described is an actinic keratosis, which is a premalignant lesion that can progress to squamous cell carcinoma.

A 75-year-old female presents to your office complaining of dizziness and hearing loss. The patient states she awoke yesterday with dizziness, which she described as feeling the room spinning. She also notes intermittent ringing in her ears. On physical exam, the patient has lateralization of her hearing loss to the unaffected ear. Rinne test shows air conduction lasts longer than bone conduction. What is the next step in helping this patient's symptoms? 1.Order a computed tomography (CT) scan to rule out acoustic neuroma. 2.Start her on high-dose Augmentin. 3.Start the patient on a low-salt, low-caffeine diet and give her meclizine for vertigo attacks. 4.Immediate referral to an ear, nose, and throat (ENT) specialist

Start the patient on a low-salt, low-caffeine diet and give her meclizine for vertigo attacks rationale: This is the treatment for symptoms of Meniere disease. You would also want to consider an ear, nose, and throat consult to rule out other etiologies of the patient's symptoms. Meniere disease is diagnosed based on history and the exclusion of other conditions as well

You prescribe Levaquin (levofloxacin) for a severe sinus infection. What is not a possible adverse reaction to this medication? 1.Achilles tendon rupture. 2.Peripheral neuropathy. 3.Nephrotoxicity. 4.Stevens-Johnson syndrome

Stevens-Johnson syndrome. rationale: Stevens-Johnson syndrome is usually caused by allopurinol and beta-lactam antibiotics

What is the most common bacterial pathogen associated with acute otitis media? 1.Streptococcus pneumoniae. 2.Haemophilus influenzae. 3.Streptococcus pyogenes. 4.Moraxella (Branhamella) catarrhalis.

Streptococcus pneumoniae rationale: This causes 40% to 50% of cases.

Which of the following should be used with all acne medications? 1.Sunscreen. 2.Oily makeup. 3.Plain soap. 4.A light alcohol wipe once a week.

Sunscreen. rationale: Sunscreen should be used with all acne medications.

Marcie just returned from Central America with traveler's diarrhea. Which is the best treatment? 1.Metronidazole (Flagyl). 2.Supportive care. 3.Quinolone antibiotics. 4.Gastric lavage.

Supportive care. rationale: Enterotoxigenic Escherichia coli (ETEC) is the most common cause of traveler's diarrhea, which occurs after ingesting contaminated food or water. It is usually self-limiting, requiring no treatment other than supportive care. It is common in developing countries. Traveler's diarrhea caused by E coli used to be frequently treated with a 3- to 5-day course of a quinolone antibiotic, such as ciprofloxacin (Cipro).

What is the most important thing a person can do to maintain healthy skin and hopefully reduce wrinkles? 1.Keep well hydrated. 2.Use sunscreen with a sun protection factor (SPF) of at least 45. 3.Avoid smoking. 4.Use mild defatted or glycerin soap

avoid smoking rationale: The most important thing a person can do to maintain healthy skin is not smoke. Smokers develop more wrinkles and have elastosis, decreased tissue perfusion and oxygenation, and an adverse exposure to free radicals on elastic tissue.

Shelby, age 14, has a blister on her arm that is filled with clear fluid. It is the result of contact with a hot iron. How do you document this? 1.Bulla. 2.Wheal. 3.Cyst. 4.Pustule

bulla rationale: A bulla is a primary skin lesion that is filled with fluid and larger than 1 cm in diameter. It is also known as a vesicle

Mildred, age 72, presents to the clinic with a blistering rash that is generalized but located mostly in skin folds and on flexural areas. She describes the course of the rash as beginning with pruritic urticarial papules that coalesced into plaques that turned dark red in about 2 weeks, followed by the development of vesicles and bullae. She tells you that the lesions are moderate to severely pruritic. During your exam, you determine the bullae are very tense and do not rupture when pressure is applied. Her daily medications include an angiotensin-converting enzyme (ACE) inhibitor, a loop diuretic, and a nonsteroidal anti-inflammatory drug (NSAID). What is your diagnosis? 1.Dermatitis herpetiformis. 2.Pemphigus vulgaris. 3.Bullous drug eruption. 4.Bullous pemphigoid

bullous pemphigoid rationale: Most cases of bullous pemphigoid occur after 60 years of age. The bullae are very tense; firm pressure on the blisters will not result in extension into the normal skin, which occurs with pemphigus vulgaris

Alexandra, age 34, was treated with oral antibiotics 2 weeks ago for a urinary tract infection. She is seen in the office today for a follow-up visit. On physical examination, the nurse practitioner notices that she has some painless, white, slightly raised patches in her mouth. This is probably caused by: 1.Herpes simplex. 2.Aphthous ulcers. 3.Candidiasis. 4.Oral cancer.

candidiasis rationale: Painless, white, slightly raised patches in a client's mouth are probably caused by candidiasis (thrush)

Justin, an obese 42-year-old, cut his right leg 3 days ago while climbing a ladder. Today his right lower leg is warm, reddened, and painful, without a sharply demarcated border. What do you suspect? 1.Diabetic neuropathy. 2.Cellulitis. 3.Peripheral vascular disease. 4.A beginning stasis ulcer.

cellulitis rationale: Cellulitis is a spreading infection of the epidermis and subcutaneous tissue that usually begins after a break in the skin. The skin of this patient's right lower leg is warm, red, and painful. Although Justin may have diabetic neuropathy, peripheral vascular disease, or a stasis ulcer, the information is not complete enough for you to suspect those conditions. The information and assessment data given fully support a diagnosis of cellulitis.

Which manifestation is noted with carbon monoxide poisoning? 1.Circumoral pallor of the lips. 2.Cherry-red lips. 3.Cyanosis of the lips. 4.Pale pink lips

cherry-red lips rationale: Cherry-red lips are a manifestation of carbon monoxide poisoning. They also occur with ketoacidosis or acidosis from aspirin poisoning

When palpating the skin over the clavicle of Norman, age 84, you notice tenting, which is: 1.Indicative of dehydration. 2.Common in thin older adults. 3.A sign of edema. 4.Indicative of scleroderma

common in thin older adults rationale: Tenting—which occurs when pinched skin remains pinched for a few moments before resuming its normal position—over the clavicle is common in thin older adults. Skin turgor is decreased with dehydration and increased with edema and scleroderma.

The most common cause of a white pupil (leukokoria or leukocoria) in a newborn is: 1.A congenital cataract. 2.Retinoblastoma. 3.Persistent hyperplastic primary vitreous. 4.Retinal detachment

congenital cataract rationale: The most common cause of a white pupil (leukokoria or leukocoria) in a newborn is a congenital cataract. The incidence may be as high as 1 in every 500 to 1000 live births, and there is usually a family history. Some infants require no treatment; however, in other cases surgery may be performed during the first few weeks of life

Cataracts are a common occurrence in patients over 60 years of age. You counsel your patient that the best cure for cataracts is: 1.Medications. 2.Dietary supplements. 3.Corrective lens surgery. 4.Optical devices.

corrective lens surgery rationale: To date, no pharmaceutical treatment proven to delay, prevent, or reverse the development of cataracts exists. The definitive management for a cataract is a surgical approach, one that removes the defective lens and replaces it with an artificial one

Samantha, age 28, is 100 lb overweight and wants to have a gastroplasty performed. In discussing this with her, you explain that by having this procedure she may: 1.Develop diarrhea. 2.Lose too much weight. 3.Develop hemorrhoids. 4.Vomit after she eats.

develop diarrhea rationale: Diarrhea is a common problem after a gastroplasty because of the induced malabsorption.

What is the most common cause of epistaxis? 1.Digital trauma. 2.Warfarin. 3.Vitamin C deficiency. 4.Hemophilia A

digital trauma rationale: Picking the nose is the most common cause of nosebleeds

You diagnose 46-year-old Mabel with viral conjunctivitis. Your treatment should include: 1.Gentamicin ophthalmic ointment. 2.Ciprofloxacin ophthalmic drops. 3.Supportive measures and lubricating drops (artificial tears). 4.Oral erythromycin for 14 days.

Supportive measures and lubricating drops rationale: Viral conjunctivitis is treated with supportive measures, including cold compresses and lubricating eye drops. Preventive measures, such as frequent handwashing, are important, as viral conjunctivitis is highly contagious

A dark-field microscopic examination is used to diagnose: 1.Scabies. 2.Leprosy. 3.Syphilis. 4.Candida infections.

Syphilis. rationale: A dark-field microscopic examination is used to diagnose syphilis. With its special condenser, a dark-field microscope causes an oblique beam of light to refract off objects too small to be seen by conventional microscopes, such as the narrow organism (Treponema pallidum) that causes syphilis.

Sandra, age 32, comes in to the clinic. She has painful joints and a distinctive rash in a butterfly distribution on her face. The rash has red papules and plaques with a fine scale. What do you suspect? 1.Lymphocytoma cutis. 2.Relapsing polychondritis. 3.Systemic lupus erythematosus. 4.An allergic reaction.

Systemic lupus erythematosus. rationale: If a client comes in to the clinic complaining of painful joints and has a distinctive rash on the face that consists of red papules and plaques with a fine scale in a butterfly distribution, suspect systemic lupus erythematosus. Acute lupus erythematosus occurs most often in young adult women. In the acute phase, the client is febrile and ill. The presence of these skin lesions in a client with neurological disease, arthritis, renal disease, or neuropsychiatric disturbances also supports the diagnosis.

Cynthia, a 31-year-old woman with a history of depression, is seen in the office today for complaints of headaches. She was recently promoted at her job, and this has caused increased stress. She describes the headache as a tightening (viselike) feeling in the temporal and nuchal areas. The pain is bilateral and tends to wax and wane. It started approximately 2 days ago and is still present. What kind of headache is she describing? 1.Classic migraine. 2.Tension headache. 3.Sinus headache. 4.Cluster headache

Tension headache rationale: Tension headache is the most common type of headache. These headaches are usually bilateral and tend to wax and wane but have a characteristic pressure/tightening (viselike grip) in the forehead, temporal, or nuchal areas and can last from minutes to weeks. Tension headache may be associated with stress, depression, or anxiety disorders

While doing a face, head, and neck examination on a 16-year-old patient, you note that the palpebral fissures are abnormally narrow. What are you examining? 1.The nasolabial folds. 2.The openings between the margins of the upper and lower eyelids. 3.The thyroid gland in relation to the trachea. 4.The distance between the trigeminal nerve branches

The openings between the margins of the upper and lower eyelids. rationale: The palpebral fissures are the openings between the margins of the upper and lower eyelids. Someone who appears to be squinting is said to have narrow palpebral fissures

What significant finding(s) in a 3-year-old child with otitis media with effusion would prompt more aggressive treatment and referral? 1.There is a change in the child's hearing threshold to greater than 25 dB.2. The child has become a fussy eater. 3.The child's speech and language skills seem slightly delayed. 4.Persistent rhinitis is present

There is a change in the child's hearing threshold to greater than 25 dB. rationale: If a child with otitis media with effusion has a change in the hearing threshold greater than 25 dB and has notable speech and language delays, more aggressive treatment is indicated. When the child's hearing examination reveals a change in the hearing threshold, it is extremely important that the provider evaluate the child's achievement of developmental milestones in speech and language. Any abnormal findings warrant referral.

Marcia, age 4, is brought in to the office by her mother. She has a sore throat, difficulty swallowing, copious oral secretions, respiratory difficulty, stridor, and a temperature of 102°F but no pharyngeal erythema or cough. What do you suspect? 1.Epiglottitis. 2.Group A beta-hemolytic streptococcal pharyngitis. 3.Tonsillitis. 4.Diphtheria

epiglottitis rationale: A symptom cluster of severe throat pain with difficulty swallowing, copious oral secretions, respiratory difficulty, stridor, and fever but without pharyngeal erythema or cough is indicative of epiglottitis.

You are the nurse practitioner caring for Martha, a 47-year-old accountant. You have made a diagnosis of acute sinusitis based on Martha's history and the fact that she complains of pain behind her eye. Which sinuses are affected? 1.Maxillary. 2.Ethmoid. 3.Frontal. 4.Sphenoid

ethmoid rationale: With ethmoid sinus problems, the pain is felt behind the eye and high on the nose

Ellie, age 42, has a seizure disorder and has been taking phenytoin (Dilantin) for years. Which supplement should she also be taking if no other problems exist? 1.Vitamin B12. 2.Iron. 3.Folic acid. 4.Calcium

folic acid rationale: Clients taking phenytoin (Dilantin) should also be taking 0.4 to 1 mg/d of folic acid because Dilantin promotes a folate deficiency.

Elizabeth, age 83, presents with a 2-day history of pain and burning in the left forehead. This morning she noticed a rash with erythematous papules at that site. What do you suspect? 1.Varicella. 2.Herpes zoster. 3.Syphilis. 4.Rubella

herpes zoster rationale: The rash of herpes zoster is distinctive, in that it appears on only one side of the body. Herpes zoster begins in a dermatomal distribution, most commonly in the thoracic, cervical, and lumbosacral areas, although it also occurs on the face. The classic presentation is when there is pain for 1-2 days preceding the eruption.

Kevin, a 56-year-old lawyer, has throbbing pain in the left eye, an irregular pupil shape, marked photophobia, and redness around the iris. What is your initial diagnosis? 1.Conjunctivitis. 2.Iritis. 3.Subconjunctival hemorrhage. 4.Acute glaucoma.

iritis rationale: If a client has throbbing pain in the eye, an irregular pupil shape, marked photophobia, and redness (a deep, dull, red halo or ciliary flush) around the iris and/or cornea, suspect iritis. An immediate referral is warranted. The client may also have blurred vision

Mrs. Johnson, a 54-year-old accountant, presents to the office with a painful red eye without discharge. You should suspect: 1.Bacterial conjunctivitis. 2.Viral conjunctivitis. 3.Allergic conjunctivitis. 4.Iritis.

iritis rationale:In a client with iritis, there is rarely a discharge.

A biopsy of a small, yellow-orange papulonodule on the eyelid will probably show: 1.Fragmented, calcified elastic tissue. 2.Mature sebaceous glands. 3.Lipid-laden cells. 4.Endothelial swelling and an infiltrate rich in plasma cells

lipid-laden cells rationale: A biopsy of a small, yellow-orange papulonodule on the eyelid will probably show lipid-laden cells. This is a description of a noneruptive xanthoma of the eyelid (xanthelasma).

Sally, age 19, presents with pain and pressure over her cheeks and discolored nasal discharge. You cannot transilluminate the sinuses. You suspect which sinus to be affected? 1.Maxillary sinus. 2.Ethmoid sinus. 3.Sphenoid sinus. 4.Frontal sinus.

maxillary sinus rationale: The maxillary sinus is the largest of the paranasal sinuses and is the most commonly affected sinus. There is usually pain and pressure over the cheek. Inability to transilluminate the cavity usually indicates a cavity filled with purulent material. Discolored nasal discharge, as well as a poor response to decongestants, may also indicate sinusitis.

A 44-year-old banker comes to your office for evaluation of a pulsating headache over the left temporal region, and he rates the pain as an 8 on a scale of 1 to 10. The pain has been constant for the past several hours and is accompanied by nausea and sensitivity to light. He has had frequent, though less severe, headaches for many years, and they are usually relieved by over-the-counter medicines. He is unclear as to a precipitating event but notes that he has had visual disturbances before each headache and he has been under a lot of stress in his job. Based on this description, what is the most likely diagnosis of this type of headache? 1.Tension. 2.Migraine. 3.Cluster. 4.Temporal arteritis.

migraine rationale: Migraines are classically preceded by an aura and accompanied by nausea, vomiting (sometimes), and photophobia. They are usually unilateral.

Jim, age 59, presents with recurrent, sharply circumscribed red papules and plaques with powdery white scale on the extensor aspects of his elbows and knees. What do you suspect? 1.Actinic keratosis. 2.Eczema. 3.Psoriasis. 4.Seborrheic dermatitis

psoriasis rationale: If a client presents with recurrent, sharply circumscribed red papules and plaques with powdery white scale on the extensor aspects of his elbows and knees, suspect psoriasis. This is a classic presentation of psoriasis. Besides the extensor aspects of the elbows and knees, it occurs frequently in the presacral area and scalp, although lesions may occur anywhere.

A 25-year-old client who plays in a band complains that he finds it difficult to understand his fellow musicians at the end of a night of performing, a problem that is compounded by the noisy environment of the club. These symptoms are most characteristic of which of the following? 1.Sensorineural loss. 2.Conductive loss. 3.Tinnitus. 4.Vertigo.

sensorineural hearing loss rationale: Sensorineural loss comes from exposure to loud noises, inner ear infections, tumors, congenital and familial disorders, and aging. The results of the Weber and Rinne tests will assist in the diagnosis

You diagnose acute epiglottitis in Sally, age 5, and immediately send her to the local emergency room. Which of the following symptoms would indicate that an airway obstruction is imminent? 1.Reddened face. 2.Screaming. 3.Grabbing her throat. 4.Stridor

stridor rationale: In a pediatric client with acute epiglottitis, a number of symptoms can indicate that airway obstruction is imminent: stridor, restlessness, nasal flaring, as well as the use of accessory muscles of respiration.

April, age 50, presents with soft, raised, yellow plaques on her eyelids at the inner canthi. She is concerned that they may be cancerous skin lesions. You tell her that they are probably: 1.Xanthelasmas. 2.Pingueculae. 3.The result of arcus senilis. 4.Actinic keratoses

xanthelasmas rationale: Xanthelasmas are soft, raised, yellow plaques on the eyelids at the inner canthi. They appear frequently in women, in their 50s. Xanthelasmas occur with both high and normal lipid levels and have no pathological significance

Simon, age 72, states that he is worried because he has a bowel movement only every third day. You respond: 1."You should have two to three stools per day." 2."You should defecate once a day." 3."You should have at least three stools per week." 4."There is no such thing as a 'normal' pattern of defecation."

There is no such thing as a normal pattern of defecation rationale: There is no such thing as a "normal" pattern of defecation. Patterns of defecation vary widely and may in part be affected by dietary habits, fluid intake, bacteria in the stool, psychological stress, or voluntary postponement of defecation. Defecating every third day could be the routine pattern for Simon. He should be questioned if this is routine for him.

Jennifer, age 32, is pregnant and has genital warts (condylomata) and would like to have them treated. What should you order? 1Benzoyl peroxide. 2.Podophyllin (Podocon-25). 3.Trichloroacetic acid. 4.Corticosteroids.

Trichloroacetic acid. rationale: Genital warts (condylomata) may be treated using liquid nitrogen cryotherapy, trichloroacetic acid, or podophyllin (Podocon-25). However, podophyllin is contraindicated in pregnancy.

Which of the following conditions produces sharp, piercing facial pain that lasts for seconds to minutes? 1.Trigeminal neuralgia. 2Temporomandibular joint (TMJ) syndrome. 3.Goiter. 4.Preauricular adenitis

Trigeminal neuralgia rationale: Trigeminal neuralgia is described as a sharp, piercing, shooting facial pain that is severe but usually lasts only a short time. The origin is cranial nerve V (CN V), the trigeminal nerve

Which of the following medications/drugs are not known to cause heartburn or dyspepsia? 1.Alcohol. 2Motrin. 3.Prednisone. 4.Tylenol.

Tylenol. rationale: Tylenol is not a medication known to cause heartburn/dyspepsia.

Which gastrointestinal disease below could theoretically be completely eradicated with a total colectomy? 1.Crohn's disease. 2.Irritable bowel syndrome. 3.Ulcerative colitis. 4.Celiac disease

Ulcerative Colitis rationale: Ulcerative colitis is a disease only of the colon. While it is not the first treatment choice, total colectomy is a treatment option that can completely resolve this problem

Louis, age 52, presents with pruritus with no rash present. He has hypertension, diabetes, and end-stage renal disease (ESRD). Which of the following would be included in the differential diagnosis? 1.Uremia from chronic renal disease. 2.Contact dermatitis. 3.Lichen planus. 4.Psoriasis

Uremia for CRD Rationale: All of the conditions listed result in pruritus. However, only uremia from chronic renal disease results in pruritus with no rash present. The other conditions—contact dermatitis, lichen planus, and psoriasis—all present with a rash.

Which oral medication might be used to treat a client with chronic cholelithiasis who is a poor candidate for surgery? 1.Ursodiol (Actigall). 2.Ibuprofen (Advil). 3.Prednisone (Deltasone). 4.Surgery is the only answer

Ursodiol (Actigall). rationale: Ursodiol (Actigall) is an oral bile acid that dissolves gallstones. For dissolution, 8 to 10 mg/kg per day is given in 2 to 3 divided doses; for prevention, 300 mg twice per day is given. The safety of its use after 24 months has not been established.

Susan is a 19-year-old college student and avid swimmer. She frequently gets swimmer's ear and asks if there is anything she can do to help prevent it other than wearing earplugs, which do not really work for her. What do you suggest? 1.Start using a cotton-tipped applicator to dry the ears after swimming. 2.Use ear drops made of a solution of equal parts alcohol and vinegar in each ear after swimming. 3.Use a hair dryer on the highest setting to dry the ears. 4.Stop swimming

Use ear drops made of a solution of equal parts alcohol and vinegar in each ear after swimming. rationale: Using ear drops made of a solution of equal parts alcohol and vinegar in each ear after swimming is effective in drying the ear canal and maintaining an acidic environment, therefore preventing a favorable medium for the growth of bacteria, the cause of swimmer's ear

Susan is a 19-year-old college student and avid swimmer. She frequently gets swimmer's ear and asks if there is anything she can do to help prevent it other than wearing earplugs, which do not really work for her. What do you suggest? 1.Start using a cotton-tipped applicator to dry the ears after swimming. 2.Use ear drops made of a solution of equal parts alcohol and vinegar in each ear after swimming. 3.Use a hair dryer on the highest setting to dry the ears. 4.Stop swimming.

Use ear drops made of a solution of equal parts alcohol and vinegar in each ear after swimming. rationale: Using ear drops made of a solution of equal parts alcohol and vinegar in each ear after swimming is effective in drying the ear canal and maintaining an acidic environment, therefore preventing a favorable medium for the growth of bacteria, the cause of swimmer's ear.

A 75-year-old male presents for a routine physical. He is obese and has no abdominal pain or recent injuries or problems. He has no complaints. He lies supine for his abdominal exam. He is nontender to palpation and has a normal exam. When he sits up you see a large mass protrude from his abdomen. It is central to his abdomen and inferior to his rib cage. It disappears when he is sitting up fully. What is the patient's diagnosis? 1.Ventral (epigastric) hernia. 2.Inguinal hernia. 3.Femoral hernia. 4.Umbilical hernia.

Ventral (epigastric) hernia. rationale: This is the hernia described in the question; most of these hernias are asymptomatic.

A 10-year-old male in 5th grade presents to the pediatric office with his mother complaining of itchy and red eyes for 1 day. The patient complains of watery drainage in both eyes, associated with repetitive itching. On physical exam, he has no fever or constitutional symptoms. His vision is normal, with no decrease in extraocular movements. The patient has a sibling that just started day care recently. He also has bilateral preauricular lymph nodes that are inflamed. What is the patient's diagnosis? 1.Viral conjunctivitis. 2.Bacterial conjunctivitis. 3.Allergic conjunctivitis. 4.Blepharitis

Viral conjunctivitis rationale: This is the classic presentation of viral conjunctivitis. The patient also has exposure to kids at school and a sibling with day care exposure.

Olive has an acute exacerbation of Crohn's disease. Which laboratory test value(s) would you expect to be decreased? 1.Sedimentation rate. 2.Liver enzyme levels. 3.Vitamin A, B complex, and C levels. 4.Bilirubin level.

Vitamin A, B complex, and C levels. rationale: Folic acid and serum levels of most vitamins, including A, B complex, C, and the fat-soluble vitamins, are decreased in Crohn's disease as a result of malabsorption.

You are teaching Harvey, age 55, about the warts on his hands. What is included in your teaching? 1.Treatment is usually effective, and most warts will not recur afterward. 2.Because warts have roots, it is difficult to remove them surgically. 3.Warts are caused by the human papillomavirus. 4.Shaving the wart may prevent its recurrence

Warts are caused by the human papillomavirus. rationale: Warts are caused by the human papillomavirus. One in four people is infected with this virus, and most warts recur despite treatment.

You are teaching Harvey, age 55, about the warts on his hands. What is included in your teaching? 1.Treatment is usually effective, and most warts will not recur afterward. 2.Because warts have roots, it is difficult to remove them surgically. 3.Warts are caused by the human papillomavirus. 4.Shaving the wart may prevent its recurrence.

Warts are caused by the human papillomavirus. rationale: Warts are caused by the human papillomavirus. One in four people is infected with this virus, and most warts recur despite treatment.

Which is the most common presenting symptom of gastric cancer? 1.Weight loss. 2.Dysphagia. 3.Hematemesis. 4.Gastrointestinal bleeding

Weight loss rationale: Weight loss is usually the presenting symptom of gastric cancer.

Mia, a 27-year-old school teacher, has a 2-day history of severe left ear pain that began after 1 week of upper respiratory infection (URI) symptoms. On physical examination, you find that she has acute otitis media (AOM). She has a severe allergy to penicillin. The most appropriate antimicrobial option for this patient is: 1.Ciprofloxacin (Cipro). 2.Azithromycin (Zithromax). 3.Amoxicillin (Amoxil). 4.Cephalexin (Keflex).

Zithromax rationale: Azithromycin is not a perfect option for treating otitis media; however, it is preferred for patients with severe penicillin allergy.

When the Weber test is performed with a tuning fork to assess hearing and there is no lateralization, the nurse practitioner should document this finding as: 1.Conductive deafness. 2.Perceptive deafness. 3.A normal finding. 4.Nerve damage

a normal finding rationale: A Weber test assesses hearing by bone conduction. With normal hearing, sound is heard equally well in both ears, meaning there is no lateralization

The nurse practitioner (NP) tells Samantha, age 52, that she has an acrochordon on her neck. What is the NP referring to? 1.A nevus. 2.A skin tag. 3.A lipoma. 4.A wart.

a skin tag ratonale: Skin tags (acrochordons) are benign overgrowths of skin commonly seen after middle age and usually found on the neck, axillae, groin, upper trunk, and eyelids.

The metabolism of which drug is not affected in Marsha, age 74? 1.Alcohol. 2.Anticonvulsants. 3.Psychotropics. 4.Oral anticoagulants

alcohol rationale: Although drug metabolism by the liver is usually impaired in older adults, the metabolism of alcohol is unchanged.

Maryann, age 28, presents to the clinic because of a rapid onset of patchy hair loss. The skin within these oval patches of hair loss is very smooth. Tapered hairs that resemble exclamation points are seen at the margin of a patch of hair loss. Based on these findings, you suspect Maryann has: 1.Alopecia areata. 2.Trichotillomania. 3.Tinea capitis. 4.Androgenetic alopecia.

alopecia areata rationale: The findings are consistent with alopecia areata, ie, nonscarring hair loss of rapid onset, the pattern of which is most commonly sharply defined round or oval patches.

A 62-year-old woman presents to your clinic with a sudden right-sided headache that is worse in her right eye. She states that her vision seems blurred, and her right pupil is dilated and slow to react. The right conjunctiva is markedly injected, and the eyeball is firm. You screen her vision and find that she is 20/30 OS and 20/30 OD. She most likely has: 1.Open-angle glaucoma. 2.Angle-closure glaucoma. 3.Herpetic conjunctivitis. 4.Diabetic retinopathy.

angle-closure glaucoma rationale: In angle-closure glaucoma, the patient presents with a sudden onset of symptoms as described in this case. This client has a visual deficit and pain as well as fullness of the affected eye. This is a medical emergency, and she should be referred immediately because, without intervention, blindness can occur within days

A Gram stain of a lesion reveals large, square-ended, gram-positive rods that grow easily on blood agar. Which diagnosis does this finding confirm? 1.Dermatophyte infection. 2.Tuberculosis (scrofuloderma). 3.Sarcoidosis. 4.Anthrax

anthrax rationale: Anthrax is diagnosed with a Gram stain revealing large, square-ended, gram-positive rods that grow easily on blood agar.

Jill, age 29, has numerous transient lesions that come and go, and she is diagnosed with urticaria. What do you order? 1.Aspirin. 2.Ibuprofen. 3.Opioids. 4.Antihistamines

antihistamines rationale: Transient urticaria requires antihistamines on a regular basis.

Tom, age 50, is complaining of an itchy rash that occurred about a half hour after putting on his leather jacket. He recalls having a slightly similar rash last year when he wore his jacket. The annular lesions are on his neck and both arms. They are erythematous, sharply circumscribed, and both flat and elevated. His voice seems a little raspy, although he states that his breathing is normal. What is your first action? 1.Order a short course of systemic corticosteroids. 2.Determine the need for 0.5 mL 1:1000 epinephrine subcutaneously. 3.Start daily antihistamines. 4.Tell Tom to get rid of his leather jacket

Determine the need for 0.5 mL 1:1000 epinephrine subcutaneously. rationale: Tom has hives. Although all the actions are appropriate, the first step is to determine the need for 0.5 mL 1:1000 epinephrine subcutaneously. With Tom's neck involvement, it is most important to determine if respiratory distress is imminent; if it is, epinephrine must be administered

Dottie brings in her infant, who has gastroesophageal reflux. What do you tell her about positioning her infant? 1."Always position infants on their back to prevent sudden infant death syndrome." 2."Rotate your infant between lying on the back and on the stomach." 3."Your infant should be placed on the left side." 4."Place your infant in whatever position she remains quiet."

"Always position infants on their back to prevent sudden infant death syndrome." rationale: Babies with gastroesophageal reflux should, from birth, be placed to sleep on their back on a firm, flat mattress that is not elevated. Elevating the sleeping surface for back-sleeping babies does not reduce reflux and is not recommended. In addition, these babies should be fed a formula thickened with rice cereal and held in an upright position for 1 hour after feeding, as gravity helps prevent reflux.

Anson tells you he thinks his antacids are causing his diarrhea. You respond: 1."Antacids contain fructose, which may not be totally absorbed, resulting in fluid being drawn into the bowel." 2."Antacids contain sorbitol or mannitol, which are sugars that aren't absorbed and can cause fluid to be drawn into the bowel." 3."Antacids contain caffeine, which decreases bowel transit time." 4."Antacids may contain magnesium, which decreases bowel transit time and may contain poorly absorbed salts that draw fluid into the bowel."

"Antacids may contain magnesium, which decreases bowel transit time and may contain poorly absorbed salts that draw fluid into the bowel." rationale: Antacids may contain magnesium, which decreases bowel transit time and may contain poorly absorbed salts that result in an osmotic draw of fluid into the bowel resulting in diarrhea.

Nausea is difficult to discern in a young child. What question might you ask to determine if a child has nausea? 1."Are you sick to your tummy?" 2."Are you hungry?" 3."Are you eating the way you normally eat?" 4."Are you nauseous?

"Are you hungry?" rationale: To elicit information concerning nausea in a young child, ask the child about hunger because a young child cannot usually differentiate between hunger and mild nausea

Marge, age 36, is planning to go skiing with her fiancé. He has warned her about frostbite, and she is wondering what to do if frostbite should occur. You know she's misunderstood the directions when she tells you which of the following? 1."I should remove wet footwear if my feet are frostbitten." 2."I should rub the area with snow." 3."I should apply firm pressure to the area with a warm hand." 4."I should place my hands in my axillae if my hands are frostbitten."

"I should rub the area with snow." rationale: Rubbing or massaging the frostbitten area, especially with snow, may cause permanent tissue damage.

You're teaching Mitch, age 18, about his tinea pedis. You know he doesn't understand your directions when he tells you which of the following? 1."I should dry between my toes every day." 2."I should wash my socks with bleach." 3."I should use an antifungal powder twice a day." 4."I should wear rubber shoes in the shower to prevent transmission to others."

"I should wear rubber shoes in the shower to prevent transmission to others." rationale: Rubber- or plastic-soled shoes can harbor the fungus and therefore should not be worn. The shower should be washed with bleach to kill the fungi

Mandy, 44, was given a diagnosis of flu 1 day ago and wants to start on the "new flu medicine" right away. What do you tell her? 1."The medication is effective only if started within the first 72 hours after symptoms begin." 2."If you treat a cold, it goes away in 7 days; if you don't treat it, it goes away in 1 week." 3."The medicine has not proven its effectiveness." 4. "I'll start you on oseltamivir (Tamiflu) today. It may shorten the course of the disease and perhaps lessen the severity of your symptoms."

"I'll start you on oseltamivir (Tamiflu) today. It may shorten the course of the disease and perhaps lessen the severity of your symptoms. rationale: Two antiviral drugs, oseltamivir (Tamiflu), a pill taken by mouth, and zanamivir (Relenza), an inhaled medication, are particularly effective against the influenza viruses that cause the flu. If the virus causing the flu is type A influenza, the client may benefit from either one of these drugs

Client teaching is an integral part of successfully treating pediculosis. Which of the following statements would you incorporate into your teaching plan? 1."It's okay to resume sharing combs, headsets, and so on after being lice-free for one month." 2."Soak your combs and brushes in rubbing alcohol for eight hours." 3."Itching may continue for up to a week after successful treatment." 4."Spraying of pesticides in the immediate environment is essential to prevent recurrence."

"Itching may continue for up to a week after successful treatment." rationale: Client education is essential when treating pediculosis. Clients should be informed that itching may continue for up to a week after successful treatment because of the slow resolution of the inflammatory reaction caused by the lice infestation.

Tina has a chronic hepatitis C infection. She asks you how to prevent its transmission. You respond: 1."Do not donate blood until one year after diagnosis." 2."Abstain from sex altogether." 3."There is no possibility of transmission through razors or toothbrushes." 4."Abstain from sex during your period."

Abstain from sex during your period rationale: Because the hepatitis C virus is transmitted in blood, including menstrual blood, clients should abstain from sex during menstruation. You might also test Tina to see which genotype her hepatitis C is to see if she is a candidate for Harvoni or Epclusa.

Ellen, a 56-year-old social worker, is seen by the nurse practitioner for complaints of fever; left-sided facial pain; moderate amounts of purulent, malodorous nasal discharge; and pain and headache when bending forward. The symptoms have been occurring for approximately 6 days. On physical assessment, there is marked redness and swelling of the nasal passages and tenderness/pain on palpation over the cheekbones. The nurse practitioner should suspect: 1.Dental abscess. 2.Acute rhinosinusitis. 3.Chronic rhinosinusitis. 4.Nasal tumor

Acute rhinosinusitis rationale: The client is exhibiting classic characteristics of acute rhinosinusitis

Mr. Clark, age 78, is being treated with timolol maleate (Timoptic) drops for his chronic open-angle glaucoma. While performing a new client history and physical, you note that he is taking other medications. Which medication would you be most concerned about? 1.Aspirin therapy as prophylaxis for heart attack. 2.Ranitidine (Zantac) for gastroesophageal reflux disease. 3.Alprazolam (Xanax), an anxiolytic. 4.Atenolol (Tenormin), a beta blocker for high blood pressure

Atenolol (Tenormin), a beta blocker for high blood pressure rationale: If a client is taking timolol maleate (Timoptic) drops for chronic open-angle glaucoma, you should be most concerned if the client is also taking atenolol (Tenormin), a beta blocker, for high blood pressure. Because timolol maleate drops are beta-adrenergic blockers, additional beta blockers can cause worsening of congestive heart failure or reactive airway disease, as well as acute delirium.

The antibiotic of choice for recurrent acute otitis media (AOM) and/or treatment failure in children is: 1.Amoxicillin (Amoxil). 2.Amoxicillin and potassium clavulanate (Augmentin). 3.Azithromycin (Zithromax). 4.Prednisone (Deltasone)

Augmentin rationale: The antibiotic of choice for recurrent AOM or treatment failure is amoxicillin and potassium clavulanate (Augmentin)

Which of the following patients would not be at risk of Candida infection? 1.A patient with a history of coronary artery disease. 2.A diabetic patient. 3.A patient requiring home antibiotics while recovering from an operation for an infected hernia. 4.A patient using a steroid regimen for asthma control

CAD rationale: Coronary artery disease doesn't increase the risk of Candida infection

To differentiate among the different diagnoses of inflammatory bowel diseases, you look at the client's histological, culture, and radiological features. Mary has transmural inflammation, granulomas, focal involvement of the colon with some skipped areas, and sparing of the rectal mucosa. What do you suspect? 1.Crohn disease. 2.Ulcerative colitis. 3.Infectious colitis. 4.Ischemic colitis.

Crohn disease. rationale: Crohn disease would show transmural inflammation, granulomas, focal involvement of the colon with some skipped areas, and sparing of the rectal mucosa. The key words are "skipped areas of mucosal involvement."

Martina, age 34, has AIDS and currently suffers from diarrhea. You suspect she has which protozoal infection of the bowel? 1.Giardiasis. 2.Amebiasis. 3.Cryptosporidiosis. 4.Escherichia coli

Cryptosporidiosis. rationale: Cryptosporidiosis, a protozoal infection of the bowel, is common in immunocompromised clients. It causes villous atrophy and mild inflammatory changes and may secrete an enterotoxin

Which presentation is most concerning for skin cancer? 1.Dark pigmentation of 1 solitary nail that has developed quickly and without trauma. 2.A 1-mm blue, round, nonpalpable discoloration of the skin that has been present since birth without change. 3.A 5-mm black mole with round, regular boarders. 4.A 2-mm brown mole that is raised 1 mm but round and regular

Dark pigmentation of 1 solitary nail that has developed quickly and without trauma. rationale: This is concerning for acral melanoma.

Zena just had a hemorrhoidectomy. You know she has not understood your teaching when she tells you she will: 1.Take a sitz bath after each bowel movement for 1 to 2 weeks after surgery. 2.Drink at least 2000 mL of fluids per day. 3.Decrease her dietary fiber for 1 month. 4.Take stool softeners as prescribed.

Decrease her dietary fiber for 1 month rationale: For clients who have just had a hemorrhoidectomy, teaching would include advising them to maintain an adequate intake of dietary fiber to maintain stool bulk.

In a 2-month-old infant with vomiting and diarrhea, the most effective way of determining a fluid deficit is to check for: 1.Decreased peripheral perfusion. 2.Hyperventilation. 3.Irritability. 4. Hyperthermia.

Decreased peripheral perfusion rationale: In a 2-month-old infant with vomiting and diarrhea, the most effective way of determining a fluid deficit is to check for decreased peripheral perfusion, dry oral mucous membranes, and sunken fontanels. The body compensates for loss of fluid by shifting the interstitial fluid into the intravascular space, thereby maintaining perfusion of vital organs. If the fluid loss continues, circulating volume is diminished and vasoconstriction occurs in the peripheral vessels, resulting in decreased perfusion

In a 2-month-old infant with vomiting and diarrhea, the most effective way of determining a fluid deficit is to check for: 1.Decreased peripheral perfusion. 2.Hyperventilation. 3.Irritability. 4.Hyperthermia.

Decreased peripheral perfusion. rationale: In a 2-month-old infant with vomiting and diarrhea, the most effective way of determining a fluid deficit is to check for decreased peripheral perfusion, dry oral mucous membranes, and sunken fontanels. The body compensates for loss of fluid by shifting the interstitial fluid into the intravascular space, thereby maintaining perfusion of vital organs. If the fluid loss continues, circulating volume is diminished and vasoconstriction occurs in the peripheral vessels, resulting in decreased perfusion.

You are examining Barbara, age 27, who presents with multiple dry, dusky red, well-localized plaques with a "stuck-on" appearance. They are 5 to 20 mm in diameter and located on her face, scalp, and external ears. You note there is atrophy, telangiectasia, depigmentation, and follicular plugging present. On examination of the scalp, there are areas of total hair loss. There is depigmented scarring of the concha of the ear. Your most likely diagnosis is: 1.Seborrheic dermatitis. 2.Discoid lupus erythematosus. 3.Psoriasis. 4.Tinea capitis.

Discoid lupus erythematosus. rationale: The lesions of discoid lupus erythematosus are dry and have a stuck-on appearance, which differentiates them from seborrheic dermatitis and psoriasis. Old lesions that have caused scarring further distinguish these lesions from seborrheic dermatitis, psoriasis, and tinea capitis. Depigmented scarring of the concha of the ear is a classic finding

A 65-year-old man presents complaining of a left-sided, deep, throbbing headache and mild fatigue. On examination, the client has a tender, tortuous temporal artery. You suspect giant cell arteritis (GCA), or temporal arteritis. What is the least invasive procedure to help with diagnosis? 1. Magnetic resonance imaging (MRI) of the head. 2.Erythrocyte sedimentation rate (ESR). 3.Electroencephalogram (EEG). 4.Otoscopy

ESR rationale: An elevated ESR—anywhere from 30 to 100 mm/h—may be seen in giant cell arteritis (temporal arteritis); however, the ESR may also be normal. The temporal artery supplies the optic nerve; if temporal arteritis is suspected due to the age of the client (50 and older) and the location and character of the pain, it is essential that a referral to a surgeon be made for immediate biopsy of the artery before damage to the optic nerve occurs. A temporal artery biopsy (TAB), an invasive procedure, is the criterion standard for diagnosing temporal arteritis

Marty, age 52, notices a bulge in his midline every time he rises from bed in the morning. You tell him that it is a ventral hernia, also known as an: 1.Inguinal hernia. 2.Epigastric hernia. 3.Umbilical hernia. 4.Incisional hernia.

Epigastric hernia. rationale: A ventral hernia, also known as an epigastric hernia, occurs along the midline between the xiphoid process and the umbilicus. The fibers along the linea alba are brought together in a patchwork-type closure; the defect exists within this decussation. As these fibers weaken, the contents can herniate through the abdomen. Epigastric hernias are 3 times more likely to occur in men than women.

Mallory brings her 4-week-old infant to the office because she noticed small, yellow-white, glistening bumps on her infant's gums. She says they look like teeth, but she is worried that they may be cancer. You diagnose these bumps as: 1.Bednar aphthae. 2.Epstein pearls. 3.Buccal tumors. 4.Exostosis.

Epstein pearls rationale:Epstein pearls are a normal finding in newborns and infants. They appear as small, yellow-white, glistening, pearly papules along the median raphe of the hard palate and on the gums. They look like teeth but are small retention cysts that disappear after a few week

Helen, age 39, comes to your clinic for generalized joint pain, especially in the knees and hands. She tells you that the pain is moderate, spreading, and symmetrical. She reports having had a sore throat and low-grade fever for a few days and then developing a rash, described as warm erythema of the cheeks, which lasted about 4 days before disappearing. You ask her if the rash on the cheeks included the nasolabial folds or circumoral skin, and she tells you that it did not cover these areas. Two days later, she developed a nonspecific macular eruption that preceded the joint pain. Her rash faded within 2 weeks, but she tells you that it comes and goes if she bathes in hot water or spends time in the sunlight. Your patient's symptoms are most consistent with a diagnosis of: 1.Rubella. 2.Erythema infectiosum. 3.Rheumatoid arthritis. 4.Scarlet fever

Erythema infectiosum. rationale: Erythema infectiosum (fifth disease) is distinguished by its erythematous, warm rash, which gives the appearance of "slapped cheeks," and it does not involve the nasolabial folds or the circumoral region. Women are more likely to have joint pain as a symptom of erythema infectiosum.

What is the most common bacterial cause of traveler's diarrhea? 1.Escherichia coli. 2.Campylobacter jejuni. 3.Salmonella. 4.Shigella.

Escherichia coli. rationale: E coli is the most common pathogen responsible for traveler's diarrhea.

Harvey just came back from Mexico. Which pathogen do you suspect is responsible for his diarrhea? 1.Enterococci. 2.Escherichia coli. 3.Klebsiella. 4.Staphylococci.

Escherichia coli. rationale: E coli is the pathogen most often responsible for traveler's diarrhea (infectious diarrhea). Other causes may include viruses, other bacteria, protozoa, or parasites.

At the clinic, you are assessing Kyle, a 4-month-old baby, for the first time and notice that both eyes are turning inward. What is this called? 1.Pseudostrabismus. 2.Strabismus. 3.Esotropia. 4.Exotropia

Esotropia rationale: Esotropia is the inward turning of the eyes

An 80-year-old woman comes in to the office with complaints of a rash on the left side of her face that is blistered and painful and accompanied by left-sided eye pain. The rash broke out 2 days ago, and she remembers being very tired and feeling feverish for a week before the rash appeared. On examination, the rash follows the trigeminal nerve on the left, and she has some scleral injection and tearing. You suspect herpes zoster ophthalmicus. Based on what you know to be complications of this disease, you explain to her that she needs: 1.Antibiotics. 2.A biopsy of the rash. 3.Immediate hospitalization. 4.Ophthalmological consultation

Ophthalmological consultation rationale: In this case, because the herpes virus seems to be along the ophthalmic branch of cranial nerve V, there is considerable risk that this client could develop permanent damage in that eye; therefore, an ophthalmological consult needs to be arranged promptly to ascertain current damage and prevent any further damage

An 80-year-old woman comes in to the office with complaints of a rash on the left side of her face that is blistered and painful and accompanied by left-sided eye pain. The rash broke out 2 days ago, and she remembers being very tired and feeling feverish for a week before the rash appeared. On examination, the rash follows the trigeminal nerve on the left, and she has some scleral injection and tearing. You suspect herpes zoster ophthalmicus. Based on what you know to be complications of this disease, you explain to her that she needs: 1.Antibiotics. 2.A biopsy of the rash. 3.Immediate hospitalization. 4.Ophthalmological consultation.

Ophthalmological consultation. rationale: In this case, because the herpes virus seems to be along the ophthalmic branch of cranial nerve V, there is considerable risk that this client could develop permanent damage in that eye; therefore, an ophthalmological consult needs to be arranged promptly to ascertain current damage and prevent any further damage.

An 80-year-old woman comes in to the office with complaints of a rash on the left side of her face that is blistered and painful and accompanied by left-sided eye pain. The rash broke out 2 days ago, and she remembers being very tired and feeling feverish for a week before the rash appeared. On examination, the rash follows the trigeminal nerve on the left, and she has some scleral injection and tearing. You suspect herpes zoster ophthalmicus. Based on what you know to be complications of this disease, you explain to her that she needs: 1.Antibiotics. 2.A biopsy of the rash. 3.Immediate hospitalization. 4.Ophthalmological consultation.

Ophthalmological consultation. rationale:In this case, because the herpes virus seems to be along the ophthalmic branch of cranial nerve V, there is considerable risk that this client could develop permanent damage in that eye; therefore, an ophthalmological consult needs to be arranged promptly to ascertain current damage and prevent any further damage.

A 70-year-old client with herpes zoster has a vesicle on the tip of the nose. This may indicate: 1.Ophthalmic zoster. 2.Herpes simplex. 3.Kaposi sarcoma. 4.Orf and milker's nodules

Opthalmic zoster rationale: Ophthalmic zoster (herpes zoster ophthalmicus) involves the ciliary body and may appear clinically as vesicles on the tip of the nose. A client with a herpetic lesion on the nose needs to be referred to an ophthalmologist to preserve the eyesight.

Kevin, age 26, has AIDS and presents to the clinic with complaints of a painful tongue covered with what look like creamy white, curdlike patches overlying erythematous mucosa. You are able to scrape off these "curds" with a tongue depressor, which assists you in making which of the following diagnoses? 1.Leukoplakia. 2.Lichen planus. 3.Oral candidiasis. 4.Oral cancer.

Oral candidiasis rationale: Oral candidiasis (thrush) is distinctive because the white areas on the tongue can be rubbed off with a tongue depressor. Thrush may be seen in denture wearers, in debilitated clients, and in those who are immunocompromised or taking corticosteroids or broad-spectrum antibiotics

A 54-year-old female presents to your primary care office for routine reevaluation for gastroesophageal reflux disease (GERD). She has been treated with diet modifications and 6 weeks of omeprazole without improvement of her symptoms. What is the next step in management of this patient's GERD? 1.Order an endoscopy. 2.Order a Helicobacter pylori blood test. 3.Try adding ranitidine to the patient's regimen. 4.Try adding bismuth to the patient's regimen.

Order an endoscopy rationale: This is the next step in treatment in order to evaluate the etiology of the patient's GERD and consider biopsy if necessary

A 54-year-old female presents to your primary care office for routine reevaluation for gastroesophageal reflux disease (GERD). She has been treated with diet modifications and 6 weeks of omeprazole without improvement of her symptoms. What is the next step in management of this patient's GERD? 1.Order an endoscopy. 2.Order a Helicobacter pylori blood test. 3.Try adding ranitidine to the patient's regimen. 4.Try adding bismuth to the patient's regimen.

Order an endoscopy. rationale: This is the next step in treatment in order to evaluate the etiology of the patient's GERD and consider biopsy if necessary.

Michael, a 25-year-old military reservist, presents to your clinic for a rash that began on his chest and has since developed into smaller lesions that are more concentrated on the lower abdomen and pubic area. In obtaining a history of the present illness, he reports that he had an upper respiratory infection 1 month before the rash developed. He tells you it started with 1 large oval-shaped lesion on his left chest, and 1 to 2 weeks later he developed numerous smaller lesions on the lower abdomen and groin. It has been 2 weeks since the smaller lesions developed, and he tells you he is concerned that the rash isn't improving. As you examine the patient, you note that the lesions are salmon-colored and have a thin collarette of scale within them. The original lesion is still present. You suspect Michael has: 1.Guttate psoriasis. 2.Tinea versicolor. 3.Secondary syphilis. 4.Pityriasis rosea

Pityriasis rosea. rationale: Pityriasis rosea is a common, self-limiting, usually asymptomatic eruption with a distinct initial lesion. This "herald patch," which appears suddenly and without symptoms, usually is on the chest or back. Secondary lesions appear 1 to 2 weeks later while the herald patch remains. The collarette scaling is another classic symptom of pityriasis rosea. The lesions usually resolve spontaneously in 4 to 12 weeks without scarring. Outbreaks have been known to occur in close quarters like military barracks and dormitories.


Conjuntos de estudio relacionados

Organic Chem spring semester chap 1

View Set

Combo with Amsco AP US History Chapter 17 and 1 other

View Set

CompTIA A+ Core 1: Activity 10-3

View Set

Microbiology Chapter 4 Questions ❤️

View Set

Chapter 7: Group Influences on Consumer Behavior

View Set

Finance 8 Personal Finance - Ch 2 Money Management Skills - Focus on Personal Finance - McGraw Hill 5 edition

View Set

The Circulatory System: Blood Vessels and Circulation

View Set